Glory 2019 Final
Glory 2019 Final
Glory 2019 Final
Section2: pediatric
145-186
Section3: obgyn
187-231
Section4: surgery
232-275
Section 1..
Medicine ..
1.Someone with HF on lasix, BB, captopril he has chronic dry cough. What is
the cause ?
A.BB
B.Captopril
C.lasix
D.drug irrelevant
N.B: ACEi side effects are dry cough and hyperkalemia and facial + tongue swelling
(angioedema).
N.B: Thiazide side effects are hypokalemia and hyperuricemia
2. has fever with chills , low back pain , his urine .leukocytes +nitrate how to
manage ?
A.Oral antibiotic
B.ivabx at home
C.admission
3. Divorced women has a baby with sickle cell anemia , she want to get
married again but scared of having baby with SCA , what she can do ?
A.karyotype for the husband
B.karyotype for herself
C.HB electrophoresis for her
D. HB electrophoresis for husband
5. Case Scenario for 40 years old female with history of travel/ trauma
presented with swelling in one leg , non-pitting edema , skin color is blue,
pulse is intact, How to manage ?
A- lymphpain
B- massage and stocking
C- Anticoagulant
6. man with HTN Lab investigation normal but CT scan shows small mass - - -
lipid change?
A.adrenalectomy
B.MRI adrenal
C.observation
N.B: This is chusing by adrenal hyperplasia which cause HTN and dyslipidemia –
7. Patient with necrotizing pancreatitis drain done and patient improved what
is the metabolic effect
A.insulin resistance
B. hypoglycaemia
C.inhibit gluconeogenesis
D. lipidolysis
8.Screen test for Diabetic found, 90% sensitive, 80% specific. WOTF is
correct?
A.80 % of patient was non diabetic or 20% have DM.
B.90 % was diabetic
9. Case Scenario For man present with mass in the abdomen, US show
calcified lesion, your Dx ?
A- Carcinoid
B- lymphoma
11. fever, abdominal pain, low back pain , Brucella, ttt 6 weeks ??
triads of brucellosis is fever which increase afternoon, back pain , sweating,
The recommended treatment for brucellosis is given combination of rifampin and
doxycycline for 6-8 weeks
12. Hepatitis c needle stick , how much the risk to get infection?
A- 0.03 %
B- 3 %
C- 30 %
13. old pt had bilateral shoulder pain now bilateral hip pain?
polymyalgia rheumatics
it is a chronic inflammatory condition affect the elderly, pt will suffer from sever pain
and stiffness in the shoulder and hip but may affect all the body
14. Celiac disease biopsy done , showing villous atrophy , What else could you
finding?
B- subtle crypt lengthening
C- high lymphocyte
Yes initial ttt of hyperkalemia is Ca gluconate to avoid cardiac complication then give
the pt insulin
23. hematemesis ?
chronic liver disease Esophageal varices
Most common cause of mild hematmesis is peptic ulcer then esophageal varices
Most common cause of severe hematmesis is esophageal varices
N.B: BA management started by SABA, then add ICS, then LABA if not controlled.
30. Pt known case of PUD diagnosed 8 years ago come with vomiting 2 weeks
ago. On exam dehydrated and abdominal Splash test +ve. What expected on
ABG?
A-metabolicacidosis
B-Metabolic alkalosis.
C-compensated
N.B: This is about Gastric outlet syndrome due to pyloric hypertrophy obstruction
complicated a long standing PUD. Suction Splash test is positive in gastric outlet
obstruction.
36. Pt has abdpain , father has hx of h pylori , her urea breath test +ve , dx ?
A-h pylori infection
38. Female with hypotention ,hypok+ , hyponNa , dark line in hand scar what
next?
A-dexasupprestion test
B-cortisollevel
C-syntchan test
40. Pt 660 years ask for cardiac assessment : ASCVD RISK 6.5 % WHAT ELSE
? A-highly sensitive CRP
B- stress echo
41. meningitisprophylaxis ?
A-oral rifampicin .
depends on age (child oral rifampicin ,If adult Ciprofloxacin )
Defect in tyrosine kinase that prevent the development of mature B cells from pre B
cells in bone marrow (B lymphocytes= total lymphocytes- T lymphocytes)
Predisposed to recurrent infections with encapsulated organisms like H. influenza, S.
pneumonia due to impaired humoral immunity
51. a scenario of cardiac temponade muffled heart sound distended neck veins
on ecg specific finding
A- diffuse st elevation
B- prdepression
57.a case of GB
syndrome with features showing ascending paralysis and hx of URTI
58.treatment of GB syndrome
A plasmapheresis
B IV immunoglobulins
60. a male pt with normal T cell count but B cells almost diminished
A brutonagammaglobulinemia
62. a 60 yr old pt fights and argues with every 1 sth related to his cousin was
also mentioned which I hv forgotten
A Alzheimer’s dementia
B vascular dementia
C Huntington disease
D frontotemporaldementia
64. in psoriasis
A clinical hx and examination is adequate to reach the diagnosis
B skin biopsy is confirmatory
67. a pt after carpel tunnel release presented with pain what to give for pain
relief A triptans
B opioids
C NSAIDs
68. H pylori pt treated with clarithro amoxicillin and omeprazole for 10 days not
improved what to do?
A same regimen for 14 days again
B switch to doxycycline metronidazole and PPI.
70. a 60 yr old lady with features of osteoarthritis and varus force was
mentioned in the scenario but there were no options of medial or lateral
meniscal tears
74. a young child newly diagnosed asthmatic mother also asthmatic father is a
chain smoker
A stop smoking in the house
B give short acting beta agonist to the child
81. in the lab show low Tsh and 3×4 اوﻵﺧﺮ٢×٤pt with diffuse goiter in us have 2
nodular once ﻟاﺴﺆال ﻻاﺧﺮhighT3 what the best managment:
A- Total thyroectomy
B- Thyroid scan
C- Fna in both nodul
D- Fnainlargestnodul
83. 17 yrs old football player present with *area macular hyperpigmentation on
the trunk and arms, the area seem even lighter with sun exposure, what is the
most appropriate treatment regimen for this patient?
A. Topical steroid cream
B. Selenium sulphate
C. Oral antibiotic
D. Topical antibiotic
- pityriasis versicolor-
89. Pre renal, post renal and renal causes of renal failure. How to diagnose
each one?
Prerenal : hyaline casts
pre renal .. usually manifested by hypotension either due to dehydration or
hemorrhage + renal artery stensois
Renal : brown muddy like casts
91. Pt complaining of black stool before 3 days and came with bright fresh
rectal bleeding, doctor do upper and lower endoscopy and reveal
sigmoidoscopy, was negative and unable to reach diagnosis, what the next
management?
A. Abdomen CT
B. Capsule endoscope
C. NGT
92. complaining of fever, rigor, nausea and vomiting, splenomegaly, blood film
negative, next step?
A.Thick blood film after 72 hours
B.Thin blood film after 48 hours
C.Serology test
98- 45 male came with persistent neck pain with diaphoresis troponin normal.
ECG unremarkable. What will you do ?
A. Repeat troponin after 6 hr
B. Give ibuprofen
^ medical treatment
Note: Best drug for hyperthyroidism in pregnancy Propylthiouracil
Radioactive Iodine
^Surgical
105-pt with distal and proximal hand joint swelling and morning stiffness is
and RF -ve no other symptoms :
A-Reactive arthritis
B-Seronegative arthritis
C-Hand OA
https://www.ama-assn.org/residents-students/usmle/usmle-step-2-which-diagnosis-
hand-pain-stiffness
108- old pt with CHD on Lisinopril and frusemide and statin what you will add:
A-losrtan
B-bisoprolol
-If DAMARD not effective we should use next line treatment anti TNF-
Note: In another question (SLE), the answer was cyclo
But we said if infliximab was there , go for it !
110- Elderly pt in ICU, with euthyroid syndrome What you’ll see in his lab ?
A-high T3 ,T4 ,reverse T3
B-high T3, T4 and low reverse T3
C-high T3, T4 and normal reverse T3
D-low T3,T4 and high reverse T3
111- pt with meningitis High protien Low glucose 70% polymorphus
A-cryptoccos
B-TB
C-viral
D-bacterial
113- pt newly dx with HTN and doctor tell him about disease and complication
and medicationt what doctor did?
A-give informational case
B-build bonding with pt
114-internal medicine resident did diabetic foot examination but she forget to
close curtains when she expose pt abdomen what the ethical principles she is
broke?
A-privacy
B-dignity
C-autonomy
D-confidenenality
116- pt has numbness in lower part of periorbital aspect and upper lip what is
the nerve injured?
A-mandible
B-infraorbital
117- female his son died before 5 days and she is crying and can not sleep
since 2 days what you will give in short course??
A-lorazepam
B- fluxtein
119- 93 year oldpt take paracetamol with codien and he has symptoms after
take what is the cause?
A-codien
B-paracetamol
121- young lady with SOB, ..etc (PE) , Unsyable ( CT shows huge emboli)
A- enaxparine
B- unfractionated heparin+ warfarin
D-altepase
124- pt on parental nutrion Lab was mentioned Only thing was elevated INR
What should you add?
A- vit K
126- pt with CVS disease and impaired lipid profile what should you give him
A-Statin
B-Fibrate
129- pt vet presented with 2 mths HX of severe lower back pain and fatigue and
now agitated , Sacal joint tendrness ,I think physical examination should
splenomegaly ,Some labs were mentioned , What is the Dx
a- Brucellosis
b- Toxoplasma
134- lady k/c of BA on short acting b agonist and steroid inhaler in the
previous yr used SABA only once
a- Stop both medications
b- Half the dose of steroid and review in 6 mths
c- Double the dose
136- Case of gout. What is the type of crystal you will find?
Explanation: (Joint aspiration and synovial fluid analysis is the only way to make a
definitive diagnosis of gout; needle-shaped and negatively birefringent monosodium
urate crystals appear in synovial fluid)
Explanation :
Treatment of acute gout usually started with NSAID, if no improvement then go to
either Colchicine or intraarticular Steroid if patient has renal failure, never use
Allopurinol in acute attack because it worsens the condition
^ Toronto
Explanation:
SIG E CAPS
Sleep (hypersomnia or insomnia)
Interest (loss of interest or pleasure inactivities)
Guilt (feelings of worthlessness orinappropriate guilt)
Energy (↓) or fatigue
Concentration (↓)
Appetite (↑ or ↓) or weight (↑ or ↓)
Psychomotor agitation or retardation
Suicidal ideation
CXR findings in neonatal lung diseases; in RDS there will be ground glass
appearance (homogeneous infiltration) + air bronchogram and decreased lung
volumes , in Meconium aspiration syndrome there will be coarse irregular patchy
infiltrates + flattening of the diaphragm + lung hyper expansion (hyperinflation) and
10-20 % may have pneumothorax)
^ Toronto
141- Clinical scenario of (painless genital ulcer + skin rash). What is the dx?
A- Secondary syphilis.
144- Young lady with presentation looks like DVT and amenorrhea for several
wks , PT -ve What is the important q to ask?
A- History OCP.
B-
152- 55 yr old male with Afib what to give him prevent complications
- Warfarin
154- pt with chronic disease came with microcytic hypochromic anemia ,fretin
double the normal dx
- Anemi of chromic disease
- Sidroplasticnemia
- IDA
168- PtoncestartvancomycinstarttogetrashWhattodo?
1- Stopandlabel
2- Continueonsmallerdose
3- Replacewithcephalosporins
Explanation:
RapidinfusionofVancomycinmayalsocauseflushingoftheupperbody
)"called "redneck" or "redmansyndrome(
169- LDL levelgoalin a normal diabetic pt
1- <70 mg/dl
2- <100 mg/dl
3- <70 mcg/dl
170- an elderly man with a history of asthma, congestive heart failure, and
peptic ulcer disease is admitted with bronchospasm and rapid atrial
fibrillation. He receives frequent nebuliser salbutamol and IV digoxin loading,
his regular medications are continued. 24 hours after admission his serum
potassium is noted to be 2.8 mmol/l.
Which of his medications is most likely caused his abnormality?
a. Digoxin
b. ACE inhibitor
c. Salbutamol
d. Ranitidine
171-Patient has pleuritic chest pain on the left side, physical examination
reveals a pleuritic fruction rub, what’s the next step
a. CT Thorax
b. Chestxray
c. Echorefertocardio
Explanation:
Usuallywedo ECH to Ruleoutthepericardialeffusion
172- a 40 year old male has been diagnosed with lung cancer, he and his
family have never smoked, has no family history of lung cancer, he works on
electricity generation.
What’s the cause?
a. Asbestos
b. Inhaledcoaldust
c. Passivesmoking
d. Radon gas
Notes:
farmerlung>>abstosis :
Cotton>>bezenosisi
Shipbulid>>asbestosis
Begozosis>>sugerchaey
173-The diagnosis of DVT is?
US duplex.
174- pt with distal and proximal hand joint swelling and morning stiffness is
and RF -ve no other symptoms?
a. Reactivearthritis
b. Seronegativearthritis
c. Hand OA
Explanation:
Weneedtoknowisitunilateralorbilateral
Weneedtoknowtheage
Thedurationofstiffness
176-Ptsmoker DM and HTN has bilateral leg pain and loss of hair what dx
a. peripheralvenusinsuffincy
b. chroniclimbischemia
c. DVT
177-old pt with CHD on lisiopril and fursamide and statin what you will add
a. losrtan
b. bisoprolol
178- ptwirh RA
onprendisolonandhydroxyureaandmethoterxateandstillhavesymptomswhatyo
uwilladd
a. azithrprion
b. infliximab
c. cyclosporin
182- A 69-year-old man is treated for chest infection. He has been on a stable
dose of warfarin for the last six months as a treatment for atrial fibrillation,
with INR recordings between 2-2.5. However, his most recent INR was 5.
Which one of the following drugs that has recently been started is likely to be
responsible for the increased INR.
1- Clarithromycin
2- Co-dydramol
3- Digoxin
4- Rifampicin
5- Temazepam
186- 6-7yrs old had clumsy gait and unable to walk or stand,hadchecken box
begore 3wks,muscle
taken no abnormalities also had resistance to neck flexion wh is the d.d
A-GBS
B-Meningoencephalities
187- Elderly pt had pneumonia she became restless and agitated for the nurse
what will do
A-Tell the family to staye her
B-Give oxy and elevate the head bed
D-Call the Dr duty and to give lorazepam 5mg
188- Pt diagnosed ALL invest wbc22 Hb8 K5.7 phosphate low uric acid high
which of the electrolyte cause this condition
A-HypoCa
B-HyperCa
C-HypoNa
D-HyperNs
189- The nearest big-city hospital is a 6-hour drive. Given your attention to
your first priority, you would now:
A. call the ambulance for immediate transport to another hospital that “knows how to
look after this thing”
B. admit the patient to the coronary care unit for observation
C. administer streptokinase or alteplase IV immediately
D. administer heparin IV immediately
E. none of the above
- note:Absolute contraindications to thrombolytic therapy include:
• Any prior intracranial hemorrhage
• Known structural cerebral vascular lesion (e.g., arteriovenous malformation)
• Known malignant intracranial neoplasm (primary or metastatic)
• Ischemic stroke within 3 months EXCEPT acute ischemic stroke within 4.5 hours
• Suspected aortic dissection
• Active bleeding or bleeding diathesis (excluding menses)
• Significant closed-head or facial trauma within 3 months
• Intracranial or intraspinal surgery within 2 months
• Severe uncontrolled hypertension (unresponsive to emergency therapy)
• For streptokinase, prior treatment within the previous 6 months
191- Which of the following statements regarding the use of heparin in patients
with AMI is (are) true?
a. heparin therapy is used almost routinely with thrombolytic therapy during the acute
phase of MI treatment, provided certain criteria are met
b. heparin is recommended whenever there is echocardiographic evidence of left
ventricular thrombi
c. heparin should be administered (unless contraindicated)
to all patients with acute anterior wall MI
d. heparin is contraindicated in patients with uncontrolled hypertension
e. all of the above
193- Which of the following is (are) true concerning aspirin in the treatment of
AMI?
a. aspirin may serve as a substitute for streptokinase
or t-PA
b. aspirin may serve as a substitute for heparin
c. aspirin may serve as a substitute for beta blockade
d. all of the above
e. none of the above
197- In this clinical case, there is a key finding on physical examination of the
patient’s abdomen that should be further assessed by:
a. abdominal ultrasonography
b. intravenous pyelography
c. digital subtraction angiography
d. computed tomography (CT)
e. magnetic resonance imaging (MRI)
- missing information
199- A 80 year old who has been in the ICU for 4 days has developed
pneuomnia. What is the most likely organism?
A. Mycoplasma pneumoniae
B. Neisesria meningitidis
C. Pseudomonas aerogenas
D. Strep pneumoniae
200- A 30 year oldfemaleis admitted for pulmonary embolism. She has a
history of 3 miscarriages. Herlupus anticoagulant test is positive. What is the
diangosis?
A. Antiphospholipid syndrome
B. Factor V Leiden
C. Hemophilia A
202- Patient has a pleuritic chest pain on the left side. Physical
examinationreveals a pleuritic fruction rub. What is the next step?
A. 21lead ecg
B. CT Thorax
C. Chest x-ray
D. Echo refer to cardio
- Initially , A
But the best is D
207-A 40 year old male has been diagnosed with lung cancer. He and
his family have never smoked, and has no family history of lung cancer. He
works in electricity generation. What is the cause?
A. Asbestos
B. Inhaled coal dust
C. Passive smoking
D. Radon gas
209- A 45-year-old woman presents to the emergency room with pleuritic chest
pain, hemoptysis and shortness of breath for the last three hours. Her
temperature is 37.9 C, blood pressure is 120/70 mmHg, pulse is 137/min and
respirations are 28/min. Her oxygen saturation is 86% on room air.Which of the
following is the most likely diagnosis?
A. Asthma
B. Myocardial infarction
C. Pulmonary edema
D. Pulmonary embolism
210- A middle-aged man present to the doctor with blurred vision and eye pain.
He was recently diagnosed with tuberculosis. What is the cause of his
symptoms?
A. Ethambutol
B. Isoniazid
C. Pyrazinamide
D. Rifampin
211- A middle-aged man present to the doctor with red urine. He was recently
diagnosed with tuberculosis. What is the cause of his symptoms?
A. Ethambutol
B. Isoniazid
C. Pyrazinamide
D. Rifampin
212- A middle-aged man present to the doctor with signs of gout . He was
recently diagnosed with tuberculosis. What is the cause of his symptoms?
A. Ethambutol
B. Isoniazid
C. Pyrazinamide
D. Rifampin
213-20 year old man presenting for a tuberculosis screen had a positive PPD.
What should the next step be?
A. Chest CT
B. Chest XR
C. Repeat PPD
D. Sputum smear
214-A 44 year old male has a chest x-ray done for insurance purposes. His
health has always been ne and he does not take any medications, he does not
have any allergies, nor does he smoke. He last saw a doctor several years ago,
but has never had a chest x-ray before. The radiologist has marked an arrow at
what appears to be a “tear shaped” body of the upper lobe parenchyma. The
work up for a malignancy has proven negative, all the labs are normal and the
physical exam is unremarkable. Based on your anatomical knowledge and
history, this most likely represents:
A. Apical pneumothorax
B. Azygos lobe
C. Consolidated pneumonia
D. Pancoast tumor
E. Tuberculosis
218-A 19-year- old female presents to the physician with malar rash, arthritis,
proteinuria, thrombocytopenia, positive ANA, and anti- dsDNA. Which of the
follwing is the most likely diagnosis?
A. Behcet's disease
B. Reactive arthritis
C. Sjogren syndrome
D. Systemic lupus erythematosus
219-A50 year old male prsents with chronic retrosternal pain, cough and a
metallic taste in the mouth. What is the most likely diagnosis?
A. Acrodermatitis
B. Angina
C. GERD
D. Gastritis
220- A patient with lung cancer has a low serum PTH and high Calcium .What
is this caused by?
A. Hyperparathyroidism
B. Hypoparathyroidism
C. PTH-related peptide
D. SIADH
223- On a routine CXR, you note a suspicious single hilar mass. The patient
has a 100 pack year history, but is asymptomatic. What is the likely diagnosis?
A. Adenocarcinoma
B. Lymph node
C. Small cell lung cancer
D. Squamous cell cancer
C is antichonilergic
233-A 62-year-old man presents to the physician with cough for the last 7
months. He also complained of hemoptysis, weight loss and constipation.
He is a chronic smoker with a 40-year history. Laboratory tests reveal
hypercalcemia. Chest x-ray shows hilar mass in the right lung.
Which of the following is the most likely diagnosis?
A. Adenocarcinoma
B. Small cell carcinoma
C. Squamous cell carcinoma
D. Tuberculosis
234-A 50 year old adult man has a postive PPD skin test. He is
asymptomatic. What should the next step be?
A. Chest XR
B. Isoniazid for 9 months
C. Isonizid and rifampin for 3 months
D. Lung biopsy
In COPD >>ibra
In asthma >> steroid
236-A 40 year old patient presents with cough during exercise. What
medication could you give her before exercise?
A. Formeterol
B. Inhaled oxygen
C. Inhaled salbutamol
238-3 year old boy presents for TB screening. His father has pulmonary
TB. His PPD is 10mm. What does this indicate?
A. Strong postive
B. Strong negative
C. Weak positive
D. Weaknegative
239. A 30 year old female has recently given birth. She presents with a
sudden onset of shortness of breath.What would you expect to see in the xray
A. Cardiomegaly
B. Increase in mediastinal width
C. Lobar infltrate
D. Pleural effusion
241-68 year old male with adenocarcioma of the lung presents with
blurring of the vision. On examination, you note ptosis and miosis. What is
the diagnosis?
A. Gustave's syndrome
B. Horner's syndrome
C. Hyponatraemia
D. Invasionof CNIII
For 6 weeks
243- A 69-year-old man is treated for chest infection. He has been on a stable
dose of warfarin for the last six months as a treatment for atrial fibrillation,
with INR recordings between 2-2.5. However, his most recent INR was 5.
Which one of the following drugs that has recently been started is likely to be
responsible for the increased INR.
1- Clarithromycin
2- Co-dydramol
3- Digoxin
4- Rifampicin
5- Temazepam
246-1-2wks Neonate has non bilious vomiting on US there is olive what you
find?
A-Hypercholaremic metabolic acidosis
B-Hyopchl Met alkalosis
C-Hypochl resp alkalosis
249-6-7yrs old had clumsy gait and unable to walk or stand,hadchecken box
before 3wks,muscle
taken no abnormalities also had resistance to neck flexion wh is the d.d
A-GBS
B-Meningoencephalities
250 -Elderly pt had pneumonia she became restless and agitated for the nurse
what will do
A-Tell the family to staye her
B-Give oxy and elevate the head bed
D-Call the Dr duty and to give lorazepam 5mg
251-Pt diagnosed ALL invest wbc22 Hb8 K5.7 phosphate low uric acid high
which of the electrolyte cause this condition
A-HypoCa
B-HyperCa
C-HypoNa
D-HyperNs
252- rheumatoid arthritis pt with swelling in hands..(not mentioned about pain)
what to advice..
A. NSAIDS
b. methotrexate will help after 3rd stage
c. intra articular corticosteroid injections
d. symptomatic tx
253- pt diabetes type 2, with no pedal pulses in left leg, cold, raised red color
swollen area over the medial side of calf…(no pic given)
a. cellulitis
b. peripheral arterial insufficiency
c. polymyositis
d. pretibial myxedema
254- Long scenario of a man with twitching of facial muscle upon tapping of
the face, there is elevation in creatinine level diagnosis:
a) Pseudo hypoparathyroidism
b) di george s/d
c) chronic renal failure
d) hypoaldosteronism
255- long scenario given about pt having hepatomegaly lab values given alt 40
ast 200 bil 160 asked about diagnosis
a. liver cirrhosis
b. hepatic angiopathy
c. portal hypertension
d. biliary cirrhosis.
256- daily requirement of vit D:
400 IU/ml
400 > less than 1 y
600 > from 1-70
800 > more than 70 y
259- 14 yr old child diagnosed with DM type 1, when to refer for ophthalmic
examination…
a. now and repeat after every 5 years
b. after every 3 years
c. each year after the age of 40.
d. yearly, after 5 years from now
DM1 > 5 y from dx then annually
DM2 > at time of dx then annually
260- pt with URTI, since 2 weeks, red congested throat pic was given…
(streptococcal pharyngitis) treatment…
a. ceftriaxone
b. vancomycin
c. doxycycline
d. cephalosporin.
270- pt exposed to mosquitoes, have high grade fever, rash appears over the
body , splenomegaly , platelet count 40,000. causative organism spread by
a. aedes egyptii
b. female anopheles
c. sand fly
d. tick
277- Younger diabetic patient came with abdominal pain, vomiting and ketones
smelled from his mouth. What is frequent cause:
a) Insulin mismanagement
b) Diet mismanagement
280- Child ate overdose of iron several hours ago, Iron level 700, best
immediate management:
a) Gastric lavage
b) Induce vomiting manually
c) Emetic drugs
d) IV Deferoxamine
282- A boy who was bitten by his brother and received tetanus shot 6 month
ago and his laceration was 1cm and you cleaned his wound next you will:
a) Give Augmentin.
b) Suture the wound.
c) Give tetanus shot
d) Send home with close observation and return in 48 hours.
284- pt sees diplopia while viewing right or left side…..where is the leision…..
A. CN 2
B. CN 3
C. CN 4
D. CN 6
286- An old man using 6 hour computer/day presented with red eye itching
foriegn body sensation reason in his eyes, came to u, ur reason for this
condition….
a- reduce tear film
b- viral keratitis
c- cataract d uveitis
287- case of bronchiectasis…..lung fibrosed and the patient very well knows
that some part of his lung is permanently damaged…what modality of
treatment r u going to offer him…
a. lung resection
b. cpap
c. steroids
d. tracheostomy
297-A patient with eye movement abnormalities and ptosis. What is the nerve
is involved?
a-3rd
b-4th
303-Which in lipid profile is the most important risk for coronary heart
disease?
A. LDL,
B. HDL,
C. Triglyceride
D. total cholesterol
306-Patient came with ECG of narrow complex tachycardia with no P-wave and
regular rhythm what most important to test?
A) TSH
B) EPS
C) Coronary angiogram
309-MI patient has big thrombus in left coronary artery what is drug that cause
throm- bolytic action?
A) TPA
B) Heparin
C) Clexane
D) Dabigatran
310-with MI he is on nitroglycerin and morphine . After 20 minutes he
got worse with raised JVP and no lung finding. BP drop.
A) Ruptured
B) Arrhythmia
C) RV infarction
315-Q about gastroenteritis.... Widal test + ve, which of the following cell will
be effected?
payers Cell
316 --Pt on ttt for meningitis with antibiotics, came later with lab result [all CBC
decreased ] ( this is a plastic anemia ?
Chloramphenicol
318-Elderly pt with pelvic pain + normal ROM pain with abdication (AVN) , your
next step ?
Radiological image
320 - 22-year-old male drug addict, visits the hospital several times with
multiple
complaints that are exaggerated. When he is not focused he seems
asymptomatic?
Malingering
323-Pt with recurrent hemartharosis , lab [factor Vlll =0.2 normal range (50-200)
what is ttt?
Factor Vlll , this sever hemophilia A (1% -5%)
Mild hemophilia, > 5% ttt davab
326-Elderly eat fibers + change in bowl habit , came with anemia microcytic
hypochromic , what is the cause ?
Change in dite [ high fibers cause iron defainanemia ]
329-Pt on antilipid drug (hyperlipemia drug) come with flushing face , with
aspirin , lab [HDL , LDL - CK] Witch drug cause this ?
Niacin
334-Relative Risk ?
A/(A+B) / C / (C+D)
335-Pt sudden fell down no breathing his pulse is raped and weak , next step?
Intapation
338-Case of TTP what is the response for it , 27 girl came with less of
consonants of seizure + hematuria + petechial rush + lab [throbocytenia] ?
ADAMTS13
339-14 yrs before one week has URTI now come with non-blanchable rash ,
what is the cause ?
Immune thrombocytopenic purpura
353-Female has painful period she use NSAID she want stronger medication
what to advice her ?
Exercise and relaxation.
354-Pt with headache same as tension headache band like + stress for one
month and use
paracetamol day after day . What is type of headache?
over use drug headache
360-Pt with symptom of otitis media redness bulging tympanic membrain, ttt?
A. Amoxicillin + calvulanic acid
364-Young pt with discharge from unilateral nose + foul odor , ur next step ?
A. X-Ray for head and chest
365-Pt with periorbital and on nose rash and painful (Dx, herpes zoster
ophthalmicus , ttt?
A. Acyclovir and refer ophtha
366-Pt with RA treated by INFH witch disense can recurrent to him when use
this drug ?
A.TB
369-Female over thinking with tachycardia when she relaxed , tachycardia and
anexiaty, what
cause ?
Norepinephrine
372-Pt DM came with hemoptysis and chest pain (infection) found to have non
septette hyphae
fungal, witch fungal non septite ?
Zygomycosis
373-Pic of swelling redness painful in upper eyelid .Dx?
Styes
375-21 year with asthma with ulcers on knee or other site Dx?
Atopic dermatitis
378-Pt on NSAID for 2 weeks come with medium and epigastric pain or upper
GI bleeding what is the cause?
Peptic ulcer
389-Complication. of strabismus?
Amblyopia
397- Elderly female came with weight loss + constipation + pelvic mass. best
initial screening?
US.
Suspected Ovarian carcinoma so the first step in any abdominal mass is US.
398- oll lady known case of hypothyroidism present with painful movement of
the right shoulder and can't raise the shoulder due to this pain. What's the
most likely diagnosis?
Adhesive capsulitis.
404- 55 Years old with AF, what u wll do to reduce the complications?
Warfarin.
407- Asthmatic c pt have no exacerepaon in last mounth, his dialy PEF is 600,
what is indicator of sevirity in this pt?
A. RR 25.
B. PFR 240.
408- Old pte admited becouse of pneumonea at night she develop restlessness
she is disoriented what u wll do physicaly to but her in her bed?
A. Call her family to set besid her.
B. Lorazepam 4 mg stat.
409- Pr presented with some coplaint, potassium wass 7.9. What u will do
next?
A. Dialysis
B. Insulin.
C. Ca gluconate.
411- Pt with URTI 3 days ago, develope shortness of breath, PND, LLedema,
troponin is high, what is apropriat step to reach the doagnosis?
A. ECHO.
B. CXR.
C. MRI.
414- Pt has cafe alue spot in his back what you will do next?
A. Council the pt about neurofibrmatosis.
B. send the patient to genteics to do genetic.
415- pt has subcutanuoes nodule and arthrlagia what one of follwoong confirm
diagnosis of Rhematic fever?
A. High crp
B. high esr
C. postive blood culture
417- Pt age of 4 years convulsing at home for 3 minute he has tonsillitis, temp
is 38 what you will do next?
A. Diazepam.
B. lorazepam.
C. amoxicillin.
D. paracetamol.
426- Patient with DLP on statin, lab results for lipid profile all within normal
range except high alanine
aminotransferee (48) ‘normal up to 40’
What you’re going to give this patient:
A- Fibrate
B- Cholestyramine
C- Ezetimibe
D- Omega 3
427-Patient with sx of gastroenteritis:
A-Rota
B-Norovirus
C-Adenovirus
Adult = nora virus
Pedia = Rota virus
ankylosing spondylitis: pain in low back and gluteal region, insidious pain with
increased pain and stiffness in the morning, pain decrease with exercise and
increase with rest.
435-Patient coming for general check-up his BP is normal and has family Hx of
cardiac diseases, what u will do for him
A - Lipid profile
436- patient coming to see the results of INR, what the drug he is taking:
A - Warfarin
437- yrs old male Patient with DM and HTN and hx of 2 stroke events in the
past. Coming with Afib. What u will do for him:
A - Warfarin to prevent any stroke event
438- yrs old male came for general check up, BP and Fast blood glucose are
within normal ranges his BMI is 28 and he is asking about ur advice:
A -Wt reduction and exercise
B - Exercise
C-Low Salt diet
441-Case of rheumatic fever (A 7yrs old boy presented with knee pain. On
examination, he has heart murmur. His parents told you that he had tonsillitis
3 wks ago and treated with paracetamol. His ASO titer is high. How u will treat:
A- Penicillin
B- steroid->-> sever carditis
C- aspirin
442- patient presented with SOB on exertion and pallor. His CBC showed low
Hb, normal MCV and high LDH
A- Hemolytic anemia
B- Microcytic
443- patient with jaundice after quinine treatment. What the deficient enzyme:
A- Glucose-6 phosphate dehydrogenase
B- Glucose dehydrogenase
C- Glucose 6 kinase
444- patient his bone marrow bx showed 80% blast cells and Auro rods:
A- AML
B- ALL
C- CML
D- CLL
AML = Auer bodies + adult + presence is myelperoxidase and esterase
ALL = in children ( Down syndrome MC )
CML = Philadelphia chromosome + LAP
CLL = asymptotic , elderly , smudge cells , CD19 , CD20
446- patient with presentation of generalized anxiety disorder, what is the best
drug for her:
A- Lorazepam
B- Bupropion
C- Buspirone
D- Citalopram > SSRI
447-Fibroid comments in
A. african race
448- Increase total cholesterol + high LDL + high triglyceride + low HDL ... ask
about the follow up after 1 year
A- Blood pressure
B- Lipid profile
451- After stopping the OCP, pt sll have amenorrhea for 6 months, gain 5 kg,
feel tired. labs show elevated glucose
A- Hypothyroidism
B- Cushing syndrome
Estrogen increases cortisol binding globulin,
453- 55 years old with atrial fibrillation, what you do to reduce complication?
Warfarin
456- Asthmatic pt with no exacerbation last month, daily PEF is 600, what is
indicator of severity in this pt?
1- RR 25
2- PFR 240
458- Pt presented with some complain potassium was 7.9 what u will do
• dialysis
• insulin
• Ca gluconate
459- Pt on dialysis develop shivering he mention history off ever 3 day ,you
sow pus drainage from the catheter what u do
• Take blood for culture and give AB
• Blood culture AB and stop dialysis 3 days
• Change the catheter
460- Pt with URT 3 days develop shortness of breath PND LL edema troponin
is high what is appropriate step to reach the diagnosis
• echo
• ecf cxr
• MRI
466- Diabetic and heavy smoker. With 100 meter . Pain when go to mosque in
left leg. During investigation. Found the right poplitibia artery insuficiant . Best
to do?
Smoking cessation
469-RA Patient with right and left knee arthritis. What the type of RA ?
If less than 5 joints so oligo (pauci) JRA
475-IBS treatment
Symptomatic If no effective give TCA antidepressants
476-Patient. With eye symptoms. Lower limb spasticity.what is the best inv to
reach diagnosis
MRI
483-Counselling to TB patient. ?
A-Contact
B-Air born
498-Smoker for long time came with couph white sputum.. FEV1 less than 70%
what is the diagnosis?
A-Chronic bronchitis
B-Bronchial asthma
C-Lung fibrosis
N.B: Chronic smoking is the most common risk factor for COPD ''90%''
N.B: If FEV1 and FVC are low so will look for the ratio;
If ratio low so COPD or BA ''obstructive pattern''
If ratio normal or high so Pulmonary fibrosis ''restrictive pattern''
500-Patient with history of chest pain since 4 week ago .increasing . In rest and
movement.diagnosis?
A-Prizemental
B-Unstable angina
C-Heberden
D-Stable angina
N.B: Chest pain that is gradually increasing and initially on exertion but now on rest
so this is unstable angina.
508-child suffer from tonsillitis, you examine the patient and you find whites
patches on it, what is the management?
A- Penicillin
509-man diagnosed with Hep C , the couple ask you what they do:
A-Abstinence
B-Use condom
C-Do intercourse with no restrict
N.B: HCV mainly transmitted by blood but also can be transmitted sexually.
511- obese has sretch marks in skin ane puffallo hump. Your diagnosis
A.chushing
B.chines
C.pheochromocytoma
512- Pt have GERD, he take PPI for one month, after that his symptoms
returned, investigation?
A- 24 h pH monitoring test
N.B: The gold standard test to confirm diagnosis of GERD is by 24 h pH test.
517- Pt with history of asthma on SABA, need to go 2nd step but she refuse
that, what you do?
A- increase does of SABA
B- add LABA
C- tell pt how take steroid and benefit
521- Long sinario Pt c/o with long time of vomiting, development of blood, no
chest pain?
1-esophageal rupture
2-Malary Weis syndrome
3-peptic ulcer
523-Pt old in ALHAJ, c/o body rash, with neck pain and fever, wbc12 hg11
lymphocyte low , how can transport this disease?
1- Airbourn
2- Droplet
525-Pt asthmatic for longtime, IUGR, and do induce of labor, what the drug
contraindication with her?
1-sintocynone
2-methocholine
3-PGF2 alpha
530- The degree of obesity in woman based on the BMI, height 154cm
weight150kg
Obese grade 3
531- The treatment of leishmaniasis?
Pentosum
532- An elderly complaining of bilateral shoulder pain and now bilateral hip
pain What is the diagnosis?
A. Rheumatoid arthritis
B. Myalgia
Polymylgia rheumatica: bilateral shoulder and hip pain
533- Patient complaining of face swelling when sleeping and it goes after he
wakes ,also he is smoker,diagnosis?
SVC obstruction due to lung cancer
535- elderly pt long scenario of heart diseases and BPH, and new diagnosis
colon cancer and mets
A. paliative care
B. need of high care
536- Pt with long senario, severe fever and neck pain and rash, blood film
wbc13 high lympocyte, what transmission of this diseases?
A. airborne
B. droplet
C. contact
537- Pt, came with fever and blood of sputum, lymph node enlargement, how
to protect another?
A. by safety clothes
B. by face mask
C. control of airdrop
D. control of contact
540-Pt came from Sudan ( with all symptoms of malaria ) blood film done and it
was -ve
What you’ll do ?
A- thin blood film
B- thick blood film again
C- thin blood film every 8 hours until 2 day
541-Case of foot hand mouth (pic) And clear symptoms . What else you’ll see
?
A- rash of foot
543- when you cover the right eye (cover test) , there is movement in the left
eye What’s the worst complication
A- nystagmus
B- strabismus
C- amblyopia
544-Female 41 yrs want to get pregnant, lab done (high LH, high FSH) More
prone to what ?
A- endometrial cancer
B- osteoporosis
C- ovarian cancer
High FSH indicates ovarian failure > menopause > low estrogen > osteoporosis
545-Case of H. Pylori (adult pt) urea breath test +ve and he on PPI +
metronidazole
What you’ll add ?
A- clarithromycin
547-Case of CP with lower limb spastic and upper limb spastic (less than
lower)
A- quadriplegic
B- hemiplegic
C- diplegia
548-71 yrs c/o urine retention, stomach distention and severe pain with
urination
A- foly catheter + culture
550-Pt diagnosed acromegaly and started ttt with octreotide what the
investigation of choice in the future?
A- Echo
B- CT abdomen
most common cause of death in such patient cardiac disease after that cance r
552-Asthmatic pt take SABA and inhaled steroid but there is no response what
is the next step
A/LABA
B/ systemic steroid
553-There is left parasternal heave and early diastolic murmur and pistol shot
at femoral artery what is the diagnosis ?
A/ Aortic regurgitation
B/ HOCM
555-Long scenario about old female see cars and other things in her plate
what is the diagnosis
A/ dilusions
B/ halucination
C/ ilusions
A visual hallucination is a perception of an external visual stimulus where none
exists. By contrast, a visual illusion is a distortion or modification of real external
visual stimuli
559-Population screening
A. Oral cancer
B. Pancreatic cancer
C. Colorectal cancer
three national population based screening programs
Breast Screen
Bowel Cancer Screening
Cervical Screening
560-23 yrs old with forearm lesion since chilhood 1×2 cm what to do
A. Laser
B. Excision
C. Radiation
D. Followup
565-pt came to ER with rash and blood in urine found sign of HSP but not
found it in investigation
(hypersensitive evasculitis)
568-GERD
A- endoscopy
B- PH depended monitor
569-(long scenario ) esophagus with metaplasia with high grade what you
give?
A- pendazole and randitadine
B- PPI
if low grade + screen 3 mo
C- Sclerotherapy, resection
577- What layer has nerves and blood vessels in the scalp
Loose Areolar CT
578- Retinoblastoma=
absence of red reflex unilateral
583- Aspirin->
Respiratory alkalosis the metabolic acidosis
586- Sickler with recurrent admissions presented with chest pain and bone
pain,management?
Reticulocytes
hydration and analgesia,
admission and analgesics
601- High JVP with basilar crepitation and lower limb edema,diagnosis?
cor pulmonale
liver cirrhosis
nephrotic
610- pt sustained only one posture flexed right arm and left is straight,dx?
Dystonia or catatonia
616- had family history of lymphoma in his brother and ....... what is the
important lymph node to be detected to diagnose lymphoma?
A/ suraclavicular lymph node
B/ periauricular lymph node
618- Woman patient with neovascular lesion in her lt leg when intervention
A. Pain
B. Claudication
C. Bad cosmotic
D. Congestive heart failure
619- pt came with fever vomiting &diarrhea rapid fluid replacement done pt
improve for awhile then she developed abnormal movement and
became comatose?
a)complicated with meningitis
b)rapid fluid replacement cause cerebral edema
620- A 73-year-old man presents pain in his right thigh. This has been getting
progressively worse for the past 9 months despite being otherwise well. An x-
ray is reported as follows:
X-ray right femur Radiolucency of subarticular region suggestive of osteolysis.
Some areas of
patchy sclerosis Bloods tests show: Calcium2.38 mmol/lPhosphate0.85
mmol/lAlkaline phosphatase544 u/LProstate specific
antigen4.4 ng/ml
What is the most appropriate action?
A. Vitamin D supplementation
B. Check serum testosterone
C. Referral to an orthopaedic surgeon
D. Referral to a urologist
E. IV bisphosphonates
622- Female pt with her husband for fertility counselling .... 31BMI
All labs are good Regular menses Your advice
A. Keep physicaly active
B. Eat less fast food
C. Include 2 portion of vegetables and grains
625- Smoker for long time Came with cough, White sputum, efv less than
70%,what is dx?
COPD
626-Female DM1 has numbness and burning sensation in feet she wish if
there's away to help her loss whight?
a-Aerobic exercise
b-Swimming
c- Jogging Traidmil
628- Patient complaining of face swelling when sleeping and it goes after he
wakes, also he is smoker, ?
bronchial carcinoma
-
-
-
-
-
-
-
-
- -
640- A 47-year-old woman presents to her primary care provider with
Raynaud's phenomenon for 3 years presents, puffy fingers, a fingertip ulcer,
and difficulty swallowing. Past medical history is unremarkable. The patient
does smoke but drinks alcohol socially. Physical examination of her hands is
shown:
641-Patient trying to quit smoking, but he couldn’t, he came to you for help:
what behavior change at this stage?
A-precontemplation
b-contemplation
C-Action
D-preparation
642- Want to stop smoking, went to hospital and asked for treatment::
action
649-Ministry of health, prevent some group of ppl to do Hajj & Umrah in 2015,
due to MERSA- Co; whose those population?
A- Lactating women
B. Young kids > 12 years
C. elderly with DM
D. HBV
650- inactive person, overweight, smoker, 44 years old man with heart
problems, What is the most common associated with heart disease:
A-Smoking
B-Obesity
C-Inactivity
658- In a village where the incidence of cretinism and iodine was less than 1
microgram the health promoters want to issue a director for the for the
management of those with cretinism.. what is the best initial management
a) TSH and t4 measurements
b) Start thyroxine medication.
c) Iodine supplementation.
659-Want to stop smoking, but now he wants to stop, which phase he is in?
A. Precontemplation
B. Contemplation
C-action
D-Preparation
663.pt with acute anterior mi ask about the best proper management ?
-asprin. Heparin . di natrate ,
-asprin . heparin
Asprin . dinatrate . thrombolysis
664-75 year old pt known HTN he developed symptoms and signs of CHF
treated well until the symptoms relieve now he is on lisinopril . frusimide .
Ask about other drug to add with the above plan
His vital normal and BP 150/9What to add
A. -bb
B. -ccb
C. spironolactone
D. -dinatrite
665- long case talk about pt diagnosed as SLE and had acute attack ask about
the management ?
A. azathioprin
B. -steroid and hydroxycloriquine
668-pt known case of asthma come for the clinic he is on inhaled B agonist
and ICS still have attack weekly what the next proper treatment ?
A. -ipratropium promide
B. -Inhaled LABA
C. -leukotriene receptor antagonist
D. -cromolyn
670-pt known COPD 65 year he is was on ICS and inhaled LABA brought by his
son disoriented c/o productive cough and fever for 3 days
ABG done , HCO3 46mmol, PCO2 50 ,Po2 4 Kba ,PH 7.27
What the best way to manage his mental state ?
A. -nasal CPaP
B. -mechnical ventilation
C. -ipratropium promide
D. -antibiotic
672-case have symptoms , signs and lab of polycythemia ask about the
diagnosis?
Polycythemia
Hyperviscosity symptoms —> headache , thrombosis ,Pruritis
673- 2chest x ray of consilidation but for acute history of cough and flue like
symptoms one talking about a SCA pt have recurrent chest infection and limb
pain ask about the prevention of this episodes and the other having empyema
what the management
Hydroxyurea iv Acute chest syndrome
676-A 28-year-old man who had had tuberculosis of the mediastinal lymph
nodes diagnosed two weeks previously and who had been started on
chemotherapy with rifampicin, isoniazid and pyrazinamide was admitted
because of the increasing dyspnoea and stridor. Chest X-ray showed
compression of both main bronchi by carinal lymph node enlargement.
What is the next step in management?
1- Start prednisolone
2. Mediastinoscopy and biopsy
3. Refer for stent insertion/tracheostomy
4. Refer for urgent CT scan of the mediastinum
5- The addition of ethambutol
677- A 28-year-old man had been treated for pulmonary tuberculosis with
rifampicin, isoniazid, pyrazinamide and ethambutol for four weeks. Pre-
treatment liver function tests were normal but his most recent investigations
revealed:
serum total bilirubin 98 micromol/l (0-18)
serum alanine aminotransferase 620u/l (5-45)
serum aspartate aminotransferase 450 u/l (5-45)
serum alkaline phosphatase 720 u/l (40-110)
Which one of the following is the most appropriate next step?
1- Stop all treatment
2. Stop ethambutol
3. Stop isoniazid
4. Stop pyrazinamide 5. Stop rifampicin
A. Asthma
B. Bronchiectasis
C. Sarcoidosis
D. Tuberculosis
679-A70 year old male has long-standing bronchiectasis. What else beside
medical treatment can benet this patient?
A. Chest physiotherapy
B. High-dose oxygen therapy
C. Rigorous exercise
D. Smoking cessation
682-40 year old female presents with an acute asthma exacerbation in the ER.
Nebulised SABA is provided. What should be given next?
A. IV SABA
B. Nebulised ipratropium
C. Oral corticosteroids
D. Theophylline
685-A 16-year-old male presents with acute severe asthma. On examination his
peripheral pulse volume fell during inspiration.
Which one of the following is the most likely explanation for this clinical sign?
1- The cardiac effect of high dose beta agonist bronchodilator drugs
2. A falling heart rate on inspiration
3. Myocardial depression due to hypoxia
4- Peripheral vasodilatation
5- Reduced left atrial filling pressure on inspiration
690-39. A 45 year old female and non-smokeris found to have a lung nodule on
CT. It appears to be composed of calcium and fat.
A. Adenocarcinoma
B. Hamartoma
C. Mystheoma
D. Squamous cell carcinoma
A. Brain MRI
B. CT for adrenal gland
694-pt diagnosed acromegaly and started ttt with octreotide what the
investigation of choice in the future ?
Echo
Ct abdomen
Echo + colonoscopy
Cuz acromegaly associated with CVD and colon cancer
695- MVA pt was fully conscious but 4 hrs later pt become unconscious and
left pupil was dilated and there was fracture at temporal bone
A. Epiduralhge
B. Subdural
C. Subarachnoid
696- .known case of chronic hepatitis b virus since two weeks he complained
of increasing fatigablity and become jaundice,all liver enzyme abnormal with
inr 1,1
A.liver biopsy
B. Ct
C. Ercp
699.pt with high grade fever and abdominal pain for 5days then become pale
and toxic
1-Single blood culture
2-Multiple blood cultures
3-Bm aspirations
4-Urine and stool cultures
700.pt febrile and palpable lymph node with white caot in tonsil, most common
complicatio
1. Pharyngitis
2. Pneumonia
3. Scarlet fever
4. GN
701.pt wih skin rash and recurrent infection, low plt , family hx of same dz in
paternal uncle
. Wichot Aldrich syndrome
702.pic of hypersegmented neutrophils and symptoms of anemia
B12 deficiency
703.pic of hypochromic microcytic anemia what else will be low besides low h
1. Mcv
2. Plt
3. Retic
705- Pt was self dependent in eating, drinking and wearing clothes. Then
he developed dementia and some abnormal behaviors and agitations.
How to manage?
A.Haloperidol
Frontotemporaldementia
711- Elderly male came to ER with SOB and difficulty sleeping at night. He
mentioned Hx of heart surgery few years ago. You did investigations and
ECHO. What is the most important thing you will be afraid of that might kill
patient?
A-Pulmonary HTN
B- Stroke
C-PE
D- Cardiogenic manifestations
712-Elderly male came with weight loss, headache, RUQ tenderness, LFT all
high. Dx?
HCC
713- Pt with Normal sodium, low potassium, and has HTN. Dx?
A-Pheochromocytoma
B- Hyperaldosteronism
715-A Case scenario of K/C DM pt came complained of fatigue, lab shows low
bicarbonate, normal Na and low k. What to give?
A-Givebucarbinfusion
B- Give IV insulin with dextrose
C- Givenormalfluid
716-A case scenario of patient K/C of non hodgkin lymphoma came with
feathers of tumor lysis syndrome. What electrolytes abnormalities you
will find?
A-HypoCa!!!
B- HyperCa!!!
717- pt with all the viral hepatitis markers negative except positive HB
Surface antibodies. Dx?
A-Previous immunization
B- Acute resolved infection
724-Pt with recurrent chest infections and low platelets. Family history
of similar condition. Dx?
Weskot Aldrich Syndrome “X linked thrombocytopenia”
727-Case of loin pain, IVU showed non opaque patches on renal pelvis, US
showed hyperdense echoic patches. Dx?
A. Renal stone
.B Tumor
C. Rena papillaeatrophy
728-Case of elderly with HCV Hx. US shows hyper-vascular mass on right lobe
of liver. Dx?
.A HCC
B. Cholangiocarcinoma
.C Hamartoma
729- Case of elderly with weight loss and obstructive jaundice (high total and
direct bilirubin) with mild liver enzymes elevation. Dx?
A. Periampullary carcinoma
730-Pt with PE, given IV Heparin for 5 days, INT 1.2, Symptoms controlled.
What to do?
A. Stop Heparin
B. Change to Enoxaparin
731-Inferior MI. What is the post emergency management (post MONA and
ECG and liver enzymes)?
A. TPA thromolysis
B. PCI with stent
.C Heparin
732- 55 years old male complained of sudden severe headache while lifting
something. What to do?
A.MRI brain and cervical spine
B.CT brain and cervical spine
High BP can lead to subarachnoid hemorrhage, heavy lifting and straining can cause
pressure rise in brain lead to aneurysm rupture .
733- Female patient diagnosed with retro peritoneal sarcoma. What is true
regarding sarcoma?
A.Transmitted to nearby lymphatics
B.Metastasis
735- 16 years old male was complaining of dysuria, after 3 days the
parents said that his urine became foul smelling. Gram negative bacilli
was found in urine. What is the causative?
A-Klebsiella
B-Proteus marbilaris
736- Croup case. Management?
A-O2!!!
B-Antibiotics
C- Mild :steriod
737- Elderly female came with dysuria and frequency. What investigation
you will do?
A-Urine analysis
B-CT abdomen
C-Something invasive
740- New test was introduced to 800 of patients who was proven th have liver
cirrhosis And other 800 who didn't have liver disease
200 of first group tested positive and 100 from 2nd group tested positive. What
is the sensitivity of this test?
A••25%
B••33%
C••57%
741- 78 years old male admitted to ICU for MI that was complicated by
pneumonia in hospital, he received Tazocin for 12 days, now complains of
right upper quadrent pain with tenderness, US was done with report of thick
gallbladder wall with fluid collection and no stones. What is the management?
A••Emergency cholecyctectomy
B••ERCP drainage
C••US guided drainage
D••Continue same medication
This is a case of acalculuscholycystitis and pt is post MI so, unfit for surgery. Pt for
percutaneous drainage. But if pt is fit the best is cholycystectomy.
742- DM, HTN with chronic heart failure, comes to clinic for medication
counseling, he takes insulin l, furosemide, enalapril, digoxin. Some tests
values where provided. K = 5.9, Glucose high, BP normal. What drug should he
stop?
A•• insulin
B•• furosemide
C•• enalapril(ACEI causes hyperkalemia)
D••digoxin
743-Child come with cola urine, pt has hx of tonsilitis, PSGN is suspected,
what test would help you confirm your diagnosis?
A Blood culture
B anti-streptolysin titer
C Complement 3 low
745-Case about some labs with high calcium and normal PTH (1.1), phosphate
was not given, renal functions were elevated slightly and asking about what is
the diagnosis?
A••Renal failure
B••2ry hyperparathyroidism
C••Primary hyperparathyroidism
D••Tertiary hyperparathyroidism
746-Case about teenager had argument with her mother and swallowed 20 pills
of acetaminophen come to emergency the second day having abd.pain and
nausea, vomiting, asking about stage?
A••Stage 1
B••Stage 2
C••Stage 3
D••Stage 4
747-Case about child with lower limb movement loss after URTI, child
describes his problem started from down and assending upward, there was
somthing about his sensation in lower limb but as I remember it wasn't lost
completly, asking about diagnosis?
A••GBS
B••Transverse mylitis
762- croup dx
763croup mx
764-case about pertusus cough
Whooping couhg>> pertussis
Barking cough>> croup
765- case about glaucoma
776-Epiglottis mx
A. Oral ABx
B. IV Abx
C. Admission
777-How to confirm TB
A. Sputum culture
778- TB precaution :
A. Contact
B. Droplet
C. Airborne
790- Pt with high SVR but all other parameters (right side, PCWP, BP, CVP).
Dx?
A. Hypovolemic shock
796- Pt with weight loss and epigastric pain (features suggests gastrin cancer i
think). What investigation you will order?
A. Endoscopy
799- Pt has diarrhea and he is smoker. His brother has crohn disease. He is
concerned about having crohn disease also. What things that might decrease
occurrence of crohn disease in this patient?
A. Smoking
B. Diet
805- Pt with features of ischemia for long time, started now to have ischemia at
rest and episodes increased. Dx?
A. Unstable angina
806- What is the thing that increased the level of BNP falsely?
811- Clinical and ECG case of Inferior MI. Pt hypotensive. Next step?
A. Right chest leads
B. ECHO
813- COPD case, best to tell you about prognosis and oxygen need in
PaO2 measurement once or twice and less than how much
824-Female came to clinice her sister which was known adult polycystic
kidney disease,she asking for screening what u will request ?
A-AbdUS
.B-Antibody for polycystic kidney disease
825-Pt came fever and sore throat on exam hyperemic tonsil wh complication
can
occur
A-Glomerulonephrites
B-Pneumonia
826- One yr old presented with high grade fever,has droling of saliva,he looks
ill,toxic,febrile what is the intervention
A-Urgent refer to ENT for tonsilectomy
B-Admit to ICU and prepare if need intubation
C-Give him Oral abs for 7 days
829- Pt developed neck pain radiated to shoulders and numbness what is the
diagnosis?
A- Rotator cuff.
B- Cervical disk prolapse.
C- Polymylagia Rheumatica.
830- Pt developed vesicle in mouth with cervical lymph node. On exam spleen
2cm. Dx?
A- EBV.
B- HSV.
C- HPV.
D- HZV.
831- Women with skin rash and malar rash and severe joint pain. CBC and RFT
normal. What to add?
A- Cyclophosphamide.
B- Mesotrexate.
C- Azathioprine.?????
833- Pt came with cervical laymph node enlargement. The patient ask to take
biopsy. WOTF if present is an indication for biopsy?
A- If LN > 1 cm.
B-If there is fever.
C- Supraclavicular LN.
846- Man with HTN developed dysuria and interrupted urine stream. PSA is 1.
Next?
A- BB.
B- Alphablocker. Prazosin
C- Transurethral prostatectomy.
847-An 8 yr old girl with weakness since birth, complaints of upper right
abdominal pain, hepatomegaly, jaundice, and ascites…diagnosis..pic of both
usg and histopathological slide
givena
a. liver cirrhosis
b. fulminant liver failure
c. budd chiarri syndrome
d. hepatic cholangipathy
849. 35 year old smoker , on examination shown white patch on the tongue,
management:
a)abx
b)no ttt
c)Close observation
d) biopsy and excision
850. A television actress is suffering from rosacea. Since she states that the
appearance will affect her career, what is your choice of treatment?
a. Oral antibiotics
b. Antihistamines
c. Topical antibiotics
d. laser
853. 40yr.old woman asking about ca cx screening, pap smear when to do and
asking for ur advice...
a. no need now
b. can be done 1 year starting from now.
c. can be done every 5 year from now
d. if 3 negative , no need to further test for screening.
855. patient with severe headache and vertigo and pain during lifting head,
cannot maintain a steady gait while walking.
a. bppv
b. vestibular neuritis
c. optic neuritis
d. snhl
857. Health worker exposed to hep b . took all vaccines earlier. what to give
now.
a. hbig + vaccine
b. hbig + lamivudine for 2 weeks
c. only lamivudine for 2 weeks
d. no risk as already immunized
858.scarlet fever treatment
A.ivig
B.erythromycin
C.intubation
D. paracetamol high dose
859- 72 years old man diagnosed with gastroenteritis and has been vomiting
for the past 72
hours. What's released in the body?
A. CRP
B. cytokines
C. apoprotien
860- patient presented with Hypotension, his phosphatase in normal level, after one
day his
phosphate level decrease, what organ damage?
A- Liver
B- kidney
C- lung
A. Sertraline
B. Paroxetine
C. Citalopram
A blood
B stool
C urea breath
863- Which muscle would be completely paralyzed by obturator nerve injury?
A. Gluteus Maximus.
B.Adductor magnus
C. Adductor longus
A. Carbamazepine
B. Prednisolone
C. Naloxon
867- 40yr old male pt with h/o syncope when he exercises and on rest amd
chest pain.on exsmntn:there s ejection systolic murmur grade 2 -4/6on the left
lower sternal border not radiating and increases when lying dwn..there is left
atrial emlargmnt.
1.Aortic stenosis
2.Pulmonic stenosis
3.HCM
4.constructve cardiomyopathy
868- Man travels to sudan 2 weeks ago , now he is presnted with fever , maleas
.... unspecific symbtom ( from 3 days ). How you will conferm diagnosis??
A) blood culture
B) Serology
A. Needle decompression .
B. Pericardiocentesis= if muffled heart sound
C. FAST ultrasound
D. Thoracotomy
870- 22-year-old male drug addict, visits the hospital several times with
multiple
complaints that are exaggerated. When he is not focused he seems
asymptomatic?
A. Somatization
B. Malingering...
873- patient presented to the ER with cough hemoptysis night sweats and
malaise. what is most appropriate initial step in the management?
874- other question was there was a patent age 60 he has hyperthyroidism he
wants to know what long term complications he'll have....?
a. Brain CT
b. Liver CT
c. Bone scan...
875- Old male present with mid clavicle mass ( smoker for 20y and I think he
drinks ) What is your initial management
A. -Core biopsy
B. -Fine needle biopsy..
C. -bronchoscopy..
A. sarcoidosis
B. berilliosis ..
C. histoplasmosis
A. 3stool parasite
B. 3 stool culture
C. concentration test
D. D-immunoassay...
878- Which drug should be stoped before IVP!?
A. Thiazides ...
B. ACEI
C. CCB..
879- dull aching headache increase with straining and coughing in DM patient
A. Viral meningitis
B. Tb meningitis
C. Brain abscess
D. Sinusit...
A. parasitemia..
B. artemether
C. quinine....
D. primaquine
A. Pneumocyte 1....
B. Pneumocyte 2
C. Reticular Interstiti
A. Aspirin
B. Penicillin...
A. cytosolic ATP
B. nuclear ATP
C. cytosolic GTP
D. nuclear GTP
A. SCC
B. adenosquamous carcinoma
C. adenocarcinoma in situ...
. young man
886- Cause of death in marfan -
887/ -Depression
seratonin defeincy
esophagus
A. esophagus
B. jujenum
C. bladder
D. descending colon
Inflammatory
by hand washing*
892/ In a study they are selecting every 10th family in the city, what isthe type
of study
Systematic study
894/ 21 yo female with chorioretinal degeneration, myopic with -8.0D what type
of myopia?
- pathological myopia
895 / MVA anterior injury of the pancreas and anterior leakage ofpancreatic
fluid will collect:
896/ Pediatric case scenario indicate respiratory distress syndrome , ask about
the deficiency:
A. dipalmitoyl
B. phosphotidylcholine*
897/ Patient trauma to obturator nerve , which muscle will be fully paralyzed :
Adductor longus
Blister
hypersensitivity pneumatis
Terminal ileum
Fentanyl
to add? : resperidone
ATN
906/ man with erectile dysfunction with normal morning erections. Where to
refer.
Pshyaiatry
Aspirin
908/ pt with fever, altered LOC for 5 days, his condition deteriorated and
developed body rash, and became jaundiced, lab tests showed low Hb, low
PLT?
TTP
909/ Thereis astudy to the effecton intensive insulin regimen in the reduction
ofneuropathy in the patient,the results are as following: Event in the regular
insulin regimen:0.092 Event in the intensive insulin regimen:0.022 Which of the
following is correct
910/ Patient was presented by back pain relieved by ambulation , what is the
best initial treatment :
physical therapy
rRNA in nucleolus
Antibiotics.
913/ Old pt, diabetic dehydrated, lab high Na low k, high bicarbonate, ketones
in urine, Dx?
A. metabolic syndrome
B. diabetic ketoacidosis
C. lactic acidosis
914/.Pain anterior to the heel Worse in the morning and better along with the
day? Diagnosis ?
Plantar fasciitis
915- loss sensation in thumb + index + ring finger What nerve injured
median
916- / Antiviral taking by inhalation?
Decongested
N.B: The midparental height is a child's expected adult height based on the heights
of the parents.
N.B: In girls; {(the father's height in cm - 13 cm) + (mother's height in cm)} / 2. N.B: In
boys; {(the father's heigh in cm) + (mother's height in cm + 13 cm)} / 2.
For example; if the father's height is 172.72 cm, mother's height is 157.48 cm, so the
midparental height for son and daughter is:
Son: 172.72 cm + (157.48 + 13 cm) / 2 = 172.72 + 170.48 = 343.2 / 2 = 171.6 cm.
Daughter: (172.72 – 13 cm) + 157.48 / 2 = 159.72 + 157.48 / 2 = 158.6 cm
2. child admitted in the ER, due to DKA severely dehydrated, managed with
fluid and insulin , bad management complication can occur ?
A.hyperkalemia
B.hypoglycemia
C. Brain edema.
D.hyperglycemia
3. Baby (2 months old ) was diagnosed with congenital heart disease came to
the ER with bluish discoloration , the mother said he was crying what you will
do?
A.immediate heart surgery
B.Reexamine after baby calmed
C.go home
D.refer to another doctor
4. 1 month year old baby has yellowish discoloration( body and eye ) he was
diagnosed with physiological jaundice. Total bilirubin 25, mainly indirect. What
is the dx?
A. Rh incomputable
B.ABO incompatible
C.c.najjar
5. 15 yrs female not menstruating yet at clinic by her parents ,she is short
stature , short neck , hypertensive. Most appropriate diagnosis?
A.Turner
B.down
C.TFS
D.normal variant
6. 3 years old baby with single palmar crease - long tongue - straight hair.
What is the dx ?
A.turner
B.Down
C.marfan
D. patue
N.B: Most common cardiac anomaly with Down is AVSD (Atrioventricular canal
malformation)..
8. Baby elevate his head when pronated, turn his head left and right coos his
age ?
A.3 months
B.4 months
9. Child less than 1 year he has vomiting and diarrhea, he is oriented aware ,
dry tongue , sunken eye , normal pulse . what is the dehydration degree ?
A.less than 1 %
B.5-9% (moderate dehydration_.
C.more than 10 %
N.B: Dehydration grades are mild, moderate, severe. N.B: Mild 3-5% volume loss =
Asymptomatic.
N.B: Moderate 6-9% volume loss = Eager to drink, dry eyes and mouth, decreased
skin turger, tachycardia, irritability, orthostatic hypotension, decreased tearing,
sunken eyes, depressed fontanelles.
N.B: Severe 10% or more volume loss = Confusion, hypotension, deep respiration,
cold mottled skin, lethargy, inability to drink.
10.Child has headache and can't concentrate on Study , Lap Hb= 7.9 MCV less
than normal , TIBC high , how to manage ?
A.oral iron
B.oral folate
C.orlavit B 12
D.packed RBCs transfusion
12. 9 year old child , her mother notice that he is having a mass on his
abdomen , she goes to the doctor and did US which shows a mass, what is the
dx?
A.wilms
13. Case Scenario for child present with Cyanosis for one hour with crying ,
Physical Examination reveals murmur in left sternal border , X-ray shows
increased pulmonary vascular marking and small hearts, what is the next step
in treatment?
A- give Abs
B- Cardiac Catheterization
C.calm the baby first (TOF case)
14. 7days newborn girl born with ambiguous genitalia with hypotension on
investigation: low Na, how to treat ?
A- Saline + glucose
B-Hydrocortisone with saline
This is CAH.
15.Case Scenario for child with nephrotic syndrome treated by steroid, what is
the first thing to improve?
A. Edema
B. protein in urine
C. urine strip for pt
16. Case Scenario for Child with DM type 1 present in E.R with symptoms and
signs of DKA, what is the next step ?
A- provide management plan
B- wait family and discuss
C- concern child for DM type 1
D- discuss with child only
20-3 yo child :
A)draws triangle
B) use stairs
21. child have abdominal pain, testes tender, horizontal testis above the other
one
A-US
B- scrotal exploration
C- angiography
2 ﻠﯿﺎﺧﺬﺳﺘﯿﺮويﺪمﻨﻔﺘﺮةطﻮيﻠﺔوﺻﺎرعﻨﺪه3.
ﺎيﺰيﻨﻨﻌﻄﯿﮭﺘﻄﻌﯿﻤﺔﻧﻌﻤﻼيﮫ؟
A- no problem to give him vaccine B- give immunoglobulins
C- give him vaccine after 1 month
N.B: Most of UH don't need any ttt. Usually the hole heals on its own by time your
child is 4 or 5 years
old. If not so do the surgery before the child enter the school.
25. 2 days old baby with seizure. LP done and reported normal. Dx?
A-neonatalsepsis
B-asphyxia
N.B: Hypoxic ischemic encephalopathy ''birth asphyxia'' is the most common cause
of neonatal seizures, with LP r/o meningitis so most likely it is birth asphyxia.
26. Child with pneumonia (in Right middle lobe) with x ray, vital signs stable.
Management?
A-admission for iv abx
B- prescribe amoxicillin for 7 days and discharge
29.Child with jaundice and anemia ,blood film , coombs direct and indirect are
positive, reticulocytes high. Dx?
A-autoimmune hemolytic anemia.
B-spherocytosis
C-laprascopic
D-observe
N.B: Best for hernia is open surgery except: female for cosmetic reason + bilateral +
obese so here it is better for lap surgery.
32. Child with abdpain , distention , diarrhea , failure to thrive , buttock muscle
wasting ?
A-celeic
B-Ibd
33. Asthmatic child mother said cannot be sure of proper technique, what the
cause of uncontrolled asthma?Bneed step up ?
A. improper technique
B. cortisollevel
C. syntchan test
36- baby after feeding spills the milk examination and labs normal
A reassurance of mother and position during feeding
-If the question only asked about Maintenance >> the answer is 1000 ml/day or 41.6
ml/h
-If the question only asked about Deficit >> the answer *could be the same as above
*coz severe dehydration means weight loss more than 10% >> the equation is
percentage of body weight loss multiply by the total weight multiply by 10 >>
10*10*10= 1000 ml/day or 41.6 ml/h
-If the question asked about maintenance and Deficit >>the answer is 1000 + 1000 =
2000 ml/day or 83.3 ml/h . Or more
39- a child with about 3 yrs hx of steatorrhea abdominal bloating and diarrhea
amylase was normal what to do?
A stool for fat
B ultrasound
C anti endomysialantibodies
40- a baby with abdominal distension and palpable bladder
A prune belly syndrome
43- Newborn 7 days with ambitigous organ with hypotension, glucose low,
sodium low. Management?
A.Saline with glucose
B.Hydrocortisone with salin
C.Saline, glucose, hydrocortisone
44- Child with nephrotic syndrome treated by steroid, what is the first indicator
for improvement?
A.Edema
B.Urine strip for protein
C. Protein in urine
46- 15years old female c/o amonerrheahtn short neck the parents below the
level what the diagnosis?
A.tuner syndrome
B.hypothyroidism
C.familial
48- 3 months infant with vomiting and recurrent chest infection on ex low
weight and failure to thrive other unremarkable. What Dx?
A. Upper esophageal sphincter hypertrophy
B. Lower esophageal sphincter hypertrophy
C. Pyloric hypertrophy
D.Trachea esophageal fistula
It might be TEF ?
49-Newborn his older brother died from immunodeficiency infections what you
do in vaccinations?
A. Give vaccinations as normal
B. Order vaccinations and ask immunology consultation
Don’t give him BCG or Do test and wait for result
50- 1 year boy came for vaccinations his older brother died of
immunodeficiency disease what you will do ?
A. Defer vaccine
B. Change IPV to OPV
51- Adolescent fight with her mother yesterday and take two packs of
acetaminophen each contain 20 tablets, came today with right lower
abdominal pain , in which phase of toxicity she is ?
A. Phase 1
B. Phase 2
C. Phase 3
D. Phase 4
53- Baby came with absent Moro reflex he is born SVD to DM mother with
weight 4.8 what’s the nerve affected?
Brachial plexus C5-C6
54- 15 years old male come to ER, LP should be done to r/o meningitis. What
to do?
A. Consent from parents
B. No consent it is an ER
C. Consent from pt
D.asset the child and take consent from parents
56- Infant at 6 weeks his/her mother is complaining of him spitting all the milk
his birth weight 3.5now its 5.5
A.Reassure
57-3 mother bring her child female with vaginal bleeding and red congested
edematous painful vulva , during the last week she passed stool without
control but she already toilet trained , what is the cause ?
a- child sexual abuse
b- other irrelevant choices i forgot them
61- 6 year girl with vulvuar itching and bleeding what you think :
A-foreign body
B-sexual abuse
62- adolescent with band like throbbing headache and he have stress at
school :
A-tension headache
B-migraine headache
63- child with coca cola urine color since 1 week and when examine him he
has congested throat with ant LN what he have :
A-acute glomeuronephritis
B-igA nephropathy
64- child with cogested throat and white patch his temp 39 c what complication
most likely he develpe?
A-pharyngitis
B-glomerunephritis
65- newborn with acute respiratory distress and he have shifted trachea what
is the management ?
A-intubation
B-bronchoscpe
C-thoractomy
D-thoracstomy
67-parent come with child not vaccinated because they think it is bad for him
what you will do?
A-consult child support
B-explains to parents about wrong myth and it is good for child
69- 3 years child fall down and vomitt 2 times and have headach but no loss of
consciousness his neurological exam is normal what you will do?
A-ct scan
B-close follow up
C-consult neurosurgery
70- mother came to ER with her child drinks paracetamol syrup she
remembered it was a small amount in bottleand his clothes have some syroup
vitally stable and he has normal appearance he spent 4 hrs in ER what it the
next?
A-observe for another 4 hrs
B-discharge with instruction
71- baby vomit large amount after feed esophaguses PH is low but his growth
chart is normal what the next?
A-observation
B-do esophageal monometry
C- abd us
72- child with cystic fibrosis and his family is normal what you will do?
A-sibling sweat chloride test
B-father sweat chloride test
73- newborn crying and refused feeding and the smell of his urine like burned
sugar what is dx?
A-pheoketonyrea
Answer : Maple syrup disease
77- 2 month baby born at 35 week his wt is 1.7 kg about his vaccinations?
A-delay for 2 month
B-give usual vaccines with usual dose
C- give usual vaccines with half dose
79- 2 month baby born at 35 week his wt is 1.7 kg about his vaccinations?
A-delay for 2 month
B-give usual vaccines with usual dose
C- give usual vaccines with half dose
80- 9 yr old with acute severe abdominal pain what is the most common cause
at this age
A- Appendicitis
E- Cholecystic
83- pt presented with hematuria and was sick and hypertensive and his
parents mentioned that he has petechia before
A-HSP
B-Post streptococcus glommuronephritis
CXR findings in neonatal lung diseases; in RDS there will be ground glass
appearance (homogeneous infiltration) + air bronchogram and decreased lung
volumes , in Meconium aspiration syndrome there will be coarse irregular patchy
infiltrates + flattening of the diaphragm + lung hyper expansion (hyperinflation) and
10-20 % may have pneumothorax)
^ Toronto
85- 12 Old months baby with gastroenteritis on oral rehydration after recovery
and recommence on normal diet start to have diarrhea. Active with mild
dehydration. TTT?
A- Milk cow free formula.
B- B- After recover use oral rehydration for 24 hrs.
C- C- For 48 hrs.
87- 9 Years old boy short, both parents below average. Dx?
A- Constitutional.
B- Familial.
88- Another q of 9 years old boy short, bone age is 6 years, both parents below
average. Dx?
A. Constitutional.
B. Familiar.
89- Delayed puberty 15 yrsshort , some problem with hair line, elevated blood
pressure, short and I think constipation. Dx?
A. Turner syndrome
B. Hypothyroidism.
91- child in DKA mum stop insulin she does not belive that he has DM which
stage she had?
Deniale stage
94-case of EBV
95- pt with Kawasaki will start him on IVIG what is the indication of poor
response to the medication?
- CRP , direct bilirubin , ALT , AST, albumin and platelet
96- child with meningitis , eyes showed papilledema
Important complication to inform the parent:
A- Visual loss
B- Hearing loss
C- Renal failure
D-Retinal hemorrhage
98- 3 yrs old fell on his head two times vomiting and c/o headache
A- Close observation
B- Head CT
..
101- child coo smile turn face follow obj when in pron position raise head
which age ?
- 4 months
105-3 mths with noisy breathing win supine, goes away when prone ??
A- Will get wors
B- Will grow out of it when reach one
C- Need surgical repair
106- friendly with strangers, can’t sit without support, play peekaboo, what’s
the developmental age of this child?
1- 6-7 months
2- 7-8 months
3- 8-9 months
4- 9-10 months
112- 8-11yrs old had night bed e out to go to toilet exam normal he feel
shamed and asked help
A-Avoid punishment
B-Alarm bed Reinforcement
C-Desmopressin intranasal
117-11yrs old had night bed e out to go to toilet exam normal he feel shamed
and asked help
A-Avoid punishment
B-Alarm bed Reinforcement
C-Desmopressin intranasal
119- 8 yr old girl with pain abdomen. she is weak since birth, h/o
hepatomegaly, with jaundice… AST 1024 ALT 600 bilirubin 370
a. biliary stricture
b. liver cirrhosis
c. hepatic cholangiopathy
d. congenital liver failure
121- Long scenario Child with Duchenne muscular Dystrophy only given
symptoms of diseases, his mother pregnant and probably fetus is a boy. How
is chance this baby to born with this diseases?
a) 12,5%
b) 25%
c) 50%
d) no any chance
122- bmi chart was given of a 14 yr old boy…asking for conclusion…bmi 32.5
a. normal wt
b. over weight
c. obese
d. morbid obese
131-Baby retate 180 and can't set , coming smile for doctor?
4-5 months
133-Baby with recurrent diarrhea + his capillary refill more than 3 sec + his BP
80/40 (sever dehydration) next step?
20ml/hr/kg bullous
134-10 year baby came with diarrhea sometime bloody + joint pain + IDA , what
is ttt Dx :
crohn's disease , RX : Aminosulicylates
136- 2 Weeks old baby with violent vomiting + epigastric mass. Next step?
Us (pyloric stenosis).
137- Ashmatic child his family stat that thy can not go out becouse the
nebuluzer machine need electicity, what u wll do to help them?
A. Tell them they are right.
B. Provide them with nebulizer with battery.
C. Clearly instruct the inhaler and tell them to stop the nebulizer.
139- peadiatric pte with bilatral testecular swlling increase when he is crrying
what to do?
A. herniotomy.
B. mish repair.
C. laproscopic mesh repair.
141- Child with Hx of stem cell transplant few months back, brought to ER by
his father because his brother developed chicken box this morning. What are
you going to do?
A. Acyclovir
B. Varicella vaccine
C. Varicella immunoglobin
D. B+C
142- Asthmatic child, his family can not go out because nebulizer machine
need electricity, what you do to help them?
1- Tell them they are right
2- Provide them with nebulizer battery
4- Clearly instruct the inhaler and tell them to stop nebulizer
148-Child above 95th centile and look larger than other child in same age .
Investigation?
Initial >> IGF1Best >> OGTT
150-Child with history of eczema in knee and flexure elbow. With maximum
cortisone dose. And the eczema extending. What to do?
Tacrolimus
152-Double bubble
Doudenal atresia
154-Down syndrome patient. with fixed split S2. And pansystolic murmur in
left sternum. What cardiac abnormality?
AVSD
155-Child sleep with milk bottle in his mouth. He develop dental Carie's.
Diagnosis?
A-Babby bottles induced tooth decay.
N.B: Babby bottle induced tooth decay is caused by the frequent long term exposure
of the child's teeth to liquids containing sugars such as milk, fruit juice and other
sweetened drinks. The sugar in these liquids pool around teeth and gums, feeding
the bacteria that cause plaque and tooth decay.
N.B: High protein diet isn't recommended for nephrotic syndrome cuz it is dangerous
and will damage the nephrons and causes renal insufficiency, so low protein intake
is recommended.
N.B: Both can be presented with abdominal pain, distension and vomiting, but
presence of susage-shaped abdominal mass on palpation + currant red jelly like
stool is suggestibve of intussusception.
162- Feel on the right side of his abdomen , mother noticed bulging what is the
diagnosis?
A. liver contusion
B. wilms
C. neuroblastoma
163- 3 years old dropped on his head, no LOC and is oriented but he vomited
twice and complaining of headache ,what will you do?
A. CT
B. MRI
C. Reassurance
164- Baby had teeth discoloring and the mother mentioned that she leaves the
bottle
On the baby’s mouth while sleeping?
A. Bottle related
B. syphilis
165- A6year old who never received vaccination except BCG at birth he is
hepB+ve ,what vaccines should he receive now?
167- neonate complain diaphragmatic hernia not cry, what do to support him
A. chest tube
168- Child admitted in ER due to DKA, severely dehydrated Managed with fluid
and insulin, bad management complications can occurs
A. hyperkalemia
B. hypoglycemia
C. brainoedema
D. hyperglycemia
171 -awasaki case ( there was no IVIG result , asking about which one of the
things if presents whould mean that the CPR high)
172-Child with Cafe au late sopts , mother said that this is birth mark in her
family.?
A- send for more genetic education
B- send for genetic to health education about neurofibromatosis
C- counseling on neurofibromatosis type 1
175--2 months baby died suddenly without any signs of abuse or fracture or
bruises, his mother is smoker and divorced
A- sudden infant death
B- maternity smoking ....
179-child complain of cervical lymph node enlargement and had family history
of lymphoma in his brother and….
What is the important lymph node to be detected to diagnose lymphoma
A-supraclavicular lymph node
B-preauricular lymph node
184-mother came with her child she is complain, there is bruises in labia
majora
A- abuse
B- Foreign body
C- trauma
188- baby with vomiting and diarrhea , looks ill cries with tears CR 3 sec
vitally stable tt?
A. Moderate dehydration give Oral fluid
B. Moderate dehydration give IV fluid
C. Sever dehydration give bolus fluid
189- A child with slightly increased Na and hypoglycemia, what fluid will you
give?
D5
193- pediatric on ventolin inher his mother notice asthmatic attack twice per
wk came for hospital one time in last month examination clear chest what to
do?
a)steroid inhaler
b)increase ventolin frequency
194- Patient e fractured humerus and ulna he cant extend his arm wrist fingers
which nerve injury
A.Ulnar
B.Median in cupital fussa
C.Medial lateral epicondil
D.Radial
195-Milestone of a baby can draw a triangle but not square :
4 years
196-Baby delivered at home presented 65 days later with rt thigh bruises other
exam unremarkable ( PT high , PTT high , other normal ) what is the dx?
A. hemorrhagic diseases of newborn
B. factor x deficiency
197-Baby delivered at home with umbilical 1-bleeding what the cause of this
bleeding?
A-factor Vlll
B- factor X
C- factor XIII
Note : if baby born in home and comes with bleeding its due to ( vit.K ) deficiency
Note: Vit k depended factors (2,7,9,10)
Note : bleeding from umbilical after clamping after birth due to ( factor XIII )
deficiency
Note : bleeding after circumcision due to ( factor VIII ) deficiency : Hemophilia A
199- Child aged 3 years old brought by his mother with episodes of
crying, fever, productive cough and drooling of saliva. 1-2 weeks ago
mother reported that her child was complaining of bilateral
conjunctivitis. Which of the following is the most causative organism?
A- Mycoplasmapneumonia
B- Adenovirus
C-Chlamydiatrichomonaspneumonitis
Viral infection from age 3 month to 5 years is the most common organism
200- Neoborn delivered at hospital, healthy, but mother reported that the
previous baby died due to immunodeficiency. What is the best action
regarding Saudi immunization program?
A-Don't give BCG
B- Give BCG with close observation
C- Referbaby
we cannot give baby BCG vaccine if there is history of immunodeficiency
202-Mother came to hospital with her healthy normal weight 6 weeks old child,
she said that with each feeding the child passing informed stool. What to do?
A- Change Milk formula
B- Giverehydration solution
203- Child 1 years old on normal formula but passing bloodless diarrhea,
vomiting and abdominal pain since. What to do?
Change formula to cow free milk
204-Child 2 months, cry 1-2 days per week, not sleep well for 1 month. Normal
weight. Otherwise healthy. Dx?
Infantile colic
205- Child diagnosed as a case of nephrotic syndrome. Anyway, now the only
complaint is generalized edema. Management?
Frusemide
209- Newborn of diabetic mother needs to take glucose infusion. Where to give
glucose through?
A. Central venous line
B. Peripheralvenous line
C. Nasogastric tube
.D Orogastrictube
210- Duringventose instrumental delivery there was an injured to
stylomastoid foreamin. What will happen to baby?
••Incomplete eye closure!!!
Bell’s palsy
212- Which of the following is true about ASD in a 6 years old boy?
A-It will close by 5 years
B-Surgery should be done at 2 years
213- 3 weeks old infant with blood drops in diapers noticed 2 times by
mother when she change diapers. Dx?
A -Juvenile polyps
B-Mickels diverticulum
215- Child is treated for eczema with topical steroid, comes to clinic with
itching and pastular lesions on top of his eczema, arrranged in grape like
pattern. What is the most likley organism that causes his superimposed
infection?
A••Herpes simplex
B••Staphylococcus aureus
C••Group A streptococcus
216- Child come with fever and headache, there is menengealirrtation sings on
examination, LP done with results. Glucose 2.9 (Normal 2.4-4.1), Protein 560
(Normal range 150-450). What ttt is best? ••Ampicellin + vancomycin!!!
A••Vancomycin + rifampin
B••Ampicill + somthing
C••Vancomycin + ceftriaxon
If bacterial, treatment according to age:
If viral : acyclovir
If TB: RIPS+ steroids 9months
217-Child come to clinic with cough, fever (38.6) for 3 days with rash started
on face then spread to trunk and white pustule on erythematous base on the
buccal mucosa (opposite the lower 1st & 2nd molars) Dx?
A••Measles (Koplik's spot)
B••Rubella
C••Scarlet fever
218-10 year old boy come with hx of diarrhea more than one month, there is
abdominal pain with blood somtimes and mucous sometimes, the child hasn't
been loosing weight, the child is reporting that he feels incomplete evacuation
even when there is nothing left in his bowel. What would be appropriate to try
first ?
••MetronidazoleCuz Amoeba infection
A••Gluten free diet
B••CT
C••Some other option
219-Child 5 years old diagnosed with tonsilitis, and asking about his brother (2
years). Prophylaxis?
A••I chose penicellin
B••Other cant remember
221-Child has flu brought by his mother for vaccinations and mother wants to
postpone because he is sick, on ex mild flow, child is active what to do
A. Reschedule in 2 wks best answer is explain to the mother then give him
B. Ignore
226- Child with features of Nisseria meningitis (similar to HSP). Asking about
what to do with his child brother?
A. Ciprofloxacine
B. Ceftriaxone
C. Rifampicin
D. Vaccination
230- Case of child with BA. Mother worried about the disease in his child in the
future. What to tell her?
A. Bronchial asthma will continue in future
B. BA will improve in future
C. BA in children is not related to BA in adult
231-Infant crying abd pain,he pass jelly and bloodystool what is the most
appropriate invest to reach the diagnosis
A-Barium enema
B-Abdxray
C-AbdUS
D-AbdCT
234- Child had fever for 5 days, erythemateous red lip and peeling of the palm
with conjuncivitis. Dx?
A- HSV.
B- Infectious mononucleosis.
C- Kawasaki disease.
235- Child had fever and dark urine. On exam there is mild tenderness in
abdomen, UA showed RBCs and +2 protein. TTT?
A- Frusimide.
B- Predinsolone.
C- Ceftriaxone.
236- 15 years old female had short stature and short neck. She didn't get her
cycle yet. Dx?
A- Turner.
237- Father brings his child due his brother died of immunodeficiency. What
vaccine should not be given in this child?
A- Influenza.
B- Inj polio.
C- Varicella.
238. developmental age of a child... can sit without support, friendly with
strangers but cannot play peek a boo....
a. 6-7 month
b. 7-8 months
c. 8-9 months
d. 9-10 months
239- Newborn with eye hemangioma that occludes the eye completely and
your concern to
not develop amblyopia when to do resection (surgery):
A. Pacifier
B. Lying in prone position
C. Position support devices
A. Consisionl
B. genetic
C. hermonal...
D. question may not complete
244- Child presented with petechiae and his platelets is 15 , otherwise healthy.
What will you
do for him?
A. Splenectomy
B. IVIG
C. Observations....
D. steroid
245- 2 weeks neonate passed unformed stool. What will you do?
A. %9-4...
B. %0
C. < %2
D. %9>
247-child fell on outstretched hand pain in distal forearm what diagnosis...
A. A.bartons
B. B.something
C. C.smith...
A. Meningitis...
B. Enephalitis
C. Orchitis
A. IM Ceftriaxone> children
250/ Pediatric case scenario indicate respiratory distress syndrome , ask about
the deficiency:
A. dipalmitoyl
B. phosphotidylcholine*
251- / Milestone baby can hold his head and when he looks at his flying hand
he laughs and coos?
A. Answer: 4m
Section3
Obgyn……
1. 48 years old female came with abnormal uterine bleeding. TVU shows sub
serous fibroid and endometrial thickness is 14 mm. Dx?
A. Fibrosarcoma
B. Myosarcoma
C. Fibromyoma
6. Postmenopausal women come to take HRT. You said she didn't need it and
no evidence to take HRT. She insisted to take it. What to do?
A. Prescribe HRT to her
B. Don't give it /Refuse
C. Consult Hospital authority/ committee
D. Refer to another Gynecologist
12. A 21-year-old woman presents to the gynecology clinic with a mass in the
left breast. She discovered this mass while showering. Her last menstrual
period was 10 days ago. There is no family history of breast cancer. On
physical exam, you palpate a 3 cm firm non-tender mass in the upper lateral
quadrant of the left breast. The mass is smooth, well circumscribed, and
mobile. There are no skin changes, nipple discharge, or axillary
lymphadenopathy. Which of the following is the most likely diagnosis in
this woman?
A. Fibroadenoma
B. Fibrocystic change
C. Intraducalpapailoma
D. Lobular carcinoma in
13. 30-year-old female pain with period and breast mass and disappear after
period, what is your next step?
Re. evaluation next period
15. Which of the following is the most common cause of hereditary breast and
ovarian cancers?
A. Alpha-fetoprotein
B. BRCA mutation ✅
C. CDH1
D. p53 mutation
16. Female patient diagnosed as a case of ovarian cancer. What is the most
important tumor marker for follow up?
A- C125
B- AFP
17. Pt with breast cancer and fibroid on tamoxifen develop vaginal bleeding US
show fibroid 8cm and hypo echogenic area what is diagnosis?
A. endometrial cancer.
B. lyomyosarcoma.
C. overian cancer.
18. Tamoxifen for breast cancer patient, she has metrorrhagia, US showed
thick endometrium, what is side effect of this drug?
A. decrease risk of DVT
B. decrease risk of pituitary adenoma
C. increase risk Endometrial cancer
D. increase risk of uterus tumor
N.B: Tamoxifen side effects: Increases risk of thrombosis and endometrial cancer
19. Female presented e scanty vaginal bleeding she noticed that postcoital
what is your action
A. Pelvic us
B. CBC
C. Assess the vagina and cervix
20. 65 years old female presented with vaginal bleeding and abd pain Ca125
very high what u will do?
A. biopsy.
B. antibiotic.
C. admit.
21. q about menopause had fibroid before 4 years 4x3 cm come now with
bleeding and in US the fibroid size 7.6 and endometrial thickness 6 (normal
<4) what is the cause?
A. Endometrial cancer
https://www.uspreventiveservicestaskforce.org/Page/Document/UpdateSumma
ryFinal/cervical-cancer-screening
N.B: Any high risk lesion on pap smear (high vaginal swab) should be followed by
colposcopy
26. You are seeing a 78 yrs old lady in clinic. She looks sick had significant
weight loss and feels tired all the time. She has pelvic mass. What is the
most appropriate screening test to confirm the diagnosis:
A. US
B. Colposcopy
C. Ca-125
N.B: Colpo for cervical ca , Endometrial biopsy for endometrial ca , US then surgical
biopsy for Ovarian ca.
28. what is the age recommended for screening by pap smear for married
women?
A. 20-24
B. 26-30
C. 30-35
D. 36-40
31. 27 years old female she did Pap smears before 3 years it was normal what
is your advice for her ?
A. reassurance
B. no pap test
C. pap test with cytology
34. Female patient with fibroid wants to keep fertility what is the tx?
A. laproscopic hestrectomy
B. laproscopic myeomectomy
C. laparotomy myeomectomy
D. laparotomy hestrectomy
39. Pregnant in her 37 week fainted and complain of severe abd pain, no
bleeding, pb 80/50, HR120, What is the diagnosis?
A. pulmonary empolism.
B. abruptio placentae
41. Pt had a previous C-S. Now pregnant at 36 weeks with abdominal pain. Vitals:
hypotensive, tachycardia. Dx?
A. Uterine rupture
B. Abruptio placenta.√
N.B: Hypotension + tachycardia are signs of abruption of the placenta.
42. 36 weeks pregnant lady has features of abruptio placenta, severe bleeding,
hypotensive, tachycardia and anemia. What to do to safe live?
A. ICU admission and
multiteam work
B. Insert 2 large bore
peripheral cannulas
and blood transfusion
C. Delivery room
N.B: The initial most important
step in Abrupto is immediate
resuscitation at ER to stabilize
patient.
45. Pregnant has abruptio placenta, what is the acid base balance of fetus
Asphyxia?
A. Res acidosis..
B. Metabolic acidosis
N.B: Pethidine cause mixed acidosis, Placenta abruptio metabolic acidosis, Pre-
eclampsia respiratory acidosis.
47. Management of
late deceleration?
A. change position.
53. Almost same question but different CTG showing no change in heart rate of
fetus during contraction, what’s the cause?
A. Oxytocin
B. Anesthesia??
C. Analgesia??
N.B: These 2 depend on CTG picture to determine type of deceleration
I had 3 ctgs
One early ...head compression can be in oxytocin.
One late ... anesthesia induced maternal hypotension lead to placentsl
hypoperfusion and fetal distress “Think abt it when the mother in epidural
anesthesia”
One was normal ctg no deceleration normal variability
If tendon reflex w hypotenstion and late deceleration mentioned = MgSO4 toxicity
54. Pt with multiple repeated contractions and sluggish fetal heart rate
response on CTG. Mother was given epidural analgesia, given oxytocin to
induce labor, given MgSO4 due preeclampsia. What is the cause of this
CTG?
A. Oxytocin
B. Analgesia
C. MgSO4
55. 34 weeks pregnant does not feel her baby movement, she goes to the
doctor and did CTG was reactive, what is your management?
A. Urgent CS
B. Bpp
C. Refer for admission
D. Reassure
57. Women pregnant 34+2 weeks had cesarean section past pregnancy due to
non-reassuring fetal monitoring, present now for external cephalic version,
US amniotic fluid index 14, responsive CTG. What would be absolute
contraindication for ecv?
A. Previous cs
B. CTG results
C. US findings
58. 43 year old she has 3 children the first child was by C -section the other 2
were vaginal birth, now she is pregnant (36 weeks) comes with Rupture of
membrane, by us placenta previa, AFI 4 cm,CX is 2 cm 50% effaced. What is
the Absolute CI for ECV ?
A. previous CS ??
B. her age
C. US result
N.B: C/I of ECV are US results of oligo or polyhydramnios + placenta previa +
previous C-S. N.B: CI of instrumental vaginal delivery is cephalopelvic disproportion.
59. Pregnant in her 38 week and 2 weeks ago she was admitted and ECV was
done for her. Currently the fetus position is in linear (or lateral not sure)
with amniotic fluid index 12. What’s the c/i for ECV in her condition?
A. Fetal position
60. 34 weeks pregnant came with breech presentation what is your next step?
A. Fu 36 weeks -ECV
67. Pregnant lady, G4p3 ,35yo, 34GA came complaining of bleeding 2hours
ago, what question will you ask next?
A. Intercourse
B. Is she satisfied with her family?
68. 34 years G4P3 GA 32 she never came to ANC visit, what is the possiple
cause?
A. visit is expensive
B. Ignorance
C. scaring from visit
69. Pregnant woman with hepatitis, what is the type of prevention when you
treat her?
A. primary
B. secondary
C. tertiary
N.B: Primary prevention = vaccination and immunization.
N.B: Secondary prevention = Screening.
N.B: Tertiary prevention = Treatment and rehabilitation to avoid complications.
72. Female in productive age , she have symptoms of DVT , your investigation
?
A. Venous duplex
B. venography
C. plethro...
D. CT angio
76. 10 weeks pregnant come with weight loss 2 kg +fatigue + dec appetite +
vomiting after each meal what is the cause of these symptoms?
A. H. Pylori
B. hyperemesis gravidarum
88. Women diabetic controlled developed dysuria and frequency urine analysis
nitrate, creatinine high, which drug is contraindicated>
A. Amoxicillin
B. Septrin (co-trimoxazole)
C. Ciprofloxacin
D. Pipracilin tazopactam
89. Pregnant 37wks came to the clinic for follow up BP140/90 no headache,
invest on proteinuria what is the
dx?
A-Pre-eclampsia
B-Chronic HTN
C-Gestational HTN
90. Pregnant women at 15 weeks presented with nausea and headache. On
exam the BP was high "168/100", otherwise healthy. Dx?
A. Preeclampsia
B. Gestational HTN
C. Pregnancy induced HTN
D. chronic HTN
97. Pregnant BP 160/110 at the end of pregnancy, what is the next step ?
MgSO4 (Missing informations)
N.B: The drug of choice to treat and prevent eclampsia is magnesium sulfate
98. Pregnant with pre-eclampsia mild 140/40 with abdominal pain .platelet and ,
uricaerd what indicate severity?
Abdominal pain
101. Pregnant, 10 weeks with HTN never went to doctor before (chronic HTN)
what is the complication?
Pre-eclampsia ( if IUGR in the option , choese it )
102. HTN in pregnancy, what would you suspect? IUGR (smoking + pregnancy
… -> IUGR)
103. (Long Case)... IUGR , witch one of the following can be the cause?
Oligohydramnios
106. Pregnant have HCV with cracked nipple , which of the following is
contraindicated to breast feeding?
A. HCV
B. cracked nipple
N.B: HCV is not CI for breastfeeding except if there is eroded or cracked nipple.
108. Pregnant lady in her first trimester, she was exposed to rubella 3 days
ago, what you’re going to do:
A. Nothing
B. MMR
C. Terminate the pregnancy
D. Rubella Igg
109. Vaccine should be given
preconception?
A. Rubella
B. Varicella
N.B: Vaccines given in preconception
care are; HBV vaccine, MMR, tetanus
and Diphtheria.
113. Pregnant lady with no past history of chicken box .best to prefect her
from disease? .
A. avoid contact with patient
B. IVIG
C. acyclovir
114. 15 yrs female not menstruating yet at clinic by her parents, she is short
stature, short neck, hypertensive. Most appropriate diagnosis?
A. Turner
B. Down
C. TFS
D. Normal variant
N.B: HTN + short stature + webbed neck + primary amenorrhea = Turner.
N.B: Conditions associated with turner syndrome; hypothyroidism + Coarctation of
aorta.
121. Pt with sudden acute pain us showed mass in ovary what is the dx?
A. Ruptured of ovarian cyst.
B. Ovarian torsion.
122. Pt with irregular menses and acne and hair in face her wt 60kg :
A. Pcos
B. Hypothyroidsm
124. Pt with irregular menses and acne and hair in face her wt 60kg :
C. Pcos
D. Hypothyroidsm
127. Case of PCOS, normal FSH, high LH, testosterone high. What investigation
to do next?
A. Glucose and lipid
B. Thyroid function
128. 30 y old women comes with painless vaginal mass under urethral orifice
that bleeds when touched, picture was provided. What is the diagnosis?
A. Bartholin abscess (wrong option)
B. Other option can't remeber
132. A female patient came to the clinic complaining of a mass on a vagina she
has a history of repeated unprotected intercourse with multiple partners, upon
examination. she has a wart in the vagina, the causative agent is:
A. Herpes simples
B. Neisseria Gonorrhoea
C. Treponemma pallidum
D. Molluscum contagiosum
133. Female pt with small papule on genital area with central pitting, h/o
unprotected sexual intercourse with multiple partners,
A. Herpes simples
B. Neisseria Gonorrhoea
209
Glory 2019 ..
C. Treponemma pallidum
D. Molluscum contagiosum
210
Glory 2019 ..
211
Glory 2019 ..
A. No follow up needed
B. Follow up Bhcg after a week and till zero.
145. A female with abdominal pain and last menses 2months back, what is your
next step?
Pregnancy test
152. After how many months you will start call a couple infertile?
12months
154. Couple came to OBGYN clinic, need to check infertility, what to assess
first?
A. start abdominal ex
B. vaginal inspection
C. general appearance
212
Glory 2019 ..
D. PV
N.B: General appearance of the couple might help you to reach diagnosis, for example:
*Hirsutism on the female supports PCOS, Lymphadenopathy or parotid enlargement in
male supports mumps orchitis.
156. Male with infertility, low testosterone, low FSH, low LH, and high prolactine?
Prolctinoma Is the diagnosis, treated by Bromocriptine (paroledl)
157. Male with infertility, Low Testosterone. Low FSH, LH and high prolactin.
What is the treatment?
Bromocriptine
159. couples with infertility ask about surrogate pregnancy u are not sure wither
allowed in Saudi or not
A. Refer them to infertility clinic
B. Warn them might not allowed
C. Give appt after few wks and review bout the topic
161. First trimester miscarriage at 5 week with history of 2nd trimester abortion
twice duo to cervical incompetence, cause now:
Cervical incompetence->2nd
Chromosomal abnormality ->1st ->11week
162. Salpingiotomy done for tubal pregnancy. After 6 weeks B-hcg high, What's
to do?
If symptomatic - laparoscopic salpingectomy
If Asymptotic - methotrexate
163. Case about 21 years old primigravida with 2 years history of infertility now
came with spotting of blood (threatened abortion case). Nex step?
213
Glory 2019 ..
164. Woman come to clinic with history of multiple abortions, now she is 11
week pregnant, what is the best predictor of complication in pregnancy?
A. Abdominal pain
B. Vaginal bleeding
C. Other options
165. Case about pregnant in 13 weeks with vaginal bleeding. US picture given of
a uterus and a sac contains something. Closed internal os. What is the type of
abortion?
A. Unembryonic sac
B. Missed
C. Threatened
166. Pt c/o of bleeding and she at 8wk gestational age on ex os closed and there
is brownish color appear during pv, the abdomen is soft, not tender or
guarding and she denies passage of any tissue By u/s you found gestational
age 7wks what is diagnosis
A. Molar pregnancy
B. Threatened abortion
C. Ectopic pregnancy
N.B: Threatened abortion Products of conception intact, intrauterine bleeding, no
dilation of cervix,
214
Glory 2019 ..
171. Pregnant at 20
week complain of
vaginal bleeding os
closed ultra sound no
fetus
Complete abortion
215
Glory 2019 ..
173. jPregnant female 10 weeks came with vaginal blew + abdominal pain a
examination gestational age 11 -12 weeks , what is the cause ?
Molar pregnant
Incomplete abortion
Bc fundal height is more
than the actual ges age
“A hydatidiform mole
presents with pain,
bleeding, and the
passage of vesicles. Ultrasound reveals a
heterogeneous uterine cystic mass with a
"snowstorm" appearance. B- hcg levels are
markedly elevated”
216
Glory 2019 ..
180. Pt 34 years developed PPH S.V.D they give Oxytocin and do massage but
not stopped what you will do?
A-Hysterectomy.
B-Ligation of uterine artery.
Treatment of PPH
Uterine massage
Nipple massage
-oxytocin ( increase contraction ) *para ventricular* and also vasopressin!!
IM 10 after 2-5 min
IV 20
IV. 40
-methergine 0.2 IM “every 2-4 hrs - contraindicated in HTN “
Carboprost! Prostaglandin f2 alpha 0.25 IM every 15 mins maximum is 8!!
“ Contraindicated in asthma “
Side effects = diarrhea
Mesoprastole = prostaglandin E1 per rectal = illegal abortion !! ( but different rout
sublingual )
Side effect = postparypyrxia
403 D
-Surgical
B linch stitch
217
Glory 2019 ..
180. A delivery before how many weeks of gestaon called the Preterm birth
refers to a delivery?
37
40
34
28
“Preterm labor Is defined as contractions and cervical changes prior to 37 weeks
gestation. Pain associated with preterm labor is intermittent as the uterus relaxes
and softens between contractions.”
218
Glory 2019 ..
185. What is the more important thing to check before instrumental delivery?
A. Breach
B. placenta abruption
C. head pelvic disproportion
186. During-instrumental delivery, the doctor crushed the left (Ithink styelohioid)
what will be affected?
Left led closure,hearing loss,loss of taste anterior2/3, loss sensation of left sid
188. The mother refused the cs that will save the baby .....
a. Refer the pt
b. Repect her wish
c. Take the husband concent
d. Do vaginal delivery
e. Do cs against her wish
189. A 20 year old pregnant women refuses CS for complete placenta previa.
Fetus is full-term and healthy. Social worker spoke to mother (imp), next step
in management?
Doctor can go to the court and get permission for CS for the sake of the baby.
190. 24 week preterm delivered , mother she is a doctor and she asked not to
Resuscitate her baby , what you will do ?
Ignore the mother and Resuscitate
191. Pt in labor, dilatation 5cm since 2 hour , effacement 100 , station 0. Management?
A-observe√
B-give oxytocin
192. Pregnant lady with 6cm and 80 effacement , ruptured members and head
station+1, what is the labor stage?
219
Glory 2019 ..
2nd
194. protein +2, ketonuria, glucose high, pregnant 37 wks + 8 days, management ?
A. induction of labor
B. CS
195. Pregnant women in 38 weeks during her C.S surgeon noted bleeding came
from the upper abdomen, what the source of bleeding?
A. liver hemangioma
B. mesentericaneurysm
C. aorticaneurysm
D. perforated peptic ulcer
N.B: jaundice = CBD stone.
198. case female pregnant Still born at 38 wks every thing normal then
discharge but after 3 days come back with bleeding from every site injection
Which best investigation ? DIC
a. Hemoglobin electrophoresis
b. Fibrin product
220
Glory 2019 ..
200. Baby brought by his mother after 7 days from delivery his weight was 3.5
and now 3.1 , why?
Normal
201. Pregnant women during vaginal delivery, what can make her has 4th
degree perineal tear ?
A. unrestrained legs and squatting position
B. unrestrained legs and sitting on chair
C. restrained legs and use of forceps and other metallic instrument
202. after delivery by 2 months, female came with urine leak from vagina during
urination. Dx?
urethrovaginal fistula
vesicovaginal fistula
203. case of a female came after 3 months of delivery, saying she used to suffer
with low mood , irritability during the first 2 weeks of delivery but symptoms
resolved now completely….what condition did she suffer from…?
a. postpartum psychosis
b. pp blues
c. pp depression
d. pp hallucinations
204. Female after delivered present with unable to breast fed her baby and no
milk at alL wtS diagnosis?
Sheehan syndrome
205. Case of Pregnancy ,did D&C , after 1 years got amenorrhea , what the
diagnosis ?
Asherman syndrome
206. PTs with Post delivery bleeding and she have asthma , what’s CI in her
case to stop bleeding?
a. Oxytocin
b. Misprostol( prostaglandins)
c. carboprost
Hemabate( carboprost) is PGF2 Alpha which can cause or aggravate bronchospasm
Note:Ergometrine is absolute contraindicated in HTN
Note: Ergometrine is relative contraindicated in asthma
Note: Carboprost (hemabate) is contraindicated in asthma
207. A woman presented with mastitis after recently giving birth to a boy, what’s
your next step in management?
221
Glory 2019 ..
209. Mother came to clinic with her infant need to council about colicky pain:
A. reassure the mother 80% of infant with same complain
B. colicky pain with flatus in abdomen
C. reassure pain relief with the 6 weeks
N.B: Infantile colick = infantile spasm = infantile dyschezia = Crying for more than
three hours a day,, fore more than 3 days a week,, for three weeks.
*It will resolve by time, most of them in first 2-3-6 months of life with no long term
effects.
210. Case of pregnant women with UTI, culture was done and result was E.coli
sensitive for TMP/SMX and ceprofloxacin and nitrofurantoin, and asking what
is the best drug to give?
A. Cipro
B. TMP/SMX
C. Nitrofurantoin
212. acute urinary urge incontance- UTI Medical treatment with antichilonergic +
physiotherapy
213. 25 year old woman was seen in the gynaeological outpatient clinic with
excessive nad offensive vaginal discharge. What organism is the most likely
cause of her vaginal discharge?
trichomonas vaginalis
222
Glory 2019 ..
214. pt with vaginal itching and bad smell the microscopic is flatellaprazite
A. trichominosis
B. BV
215. Female come with grayish vaginal discharge + PH Dx, bacterial vaginosis
what is ttt ?
Oral metronidazole
trichomonas vaginalis
218. pregnant lady complain foul smelling vaginal diacharg what u wll give?
A. Amoxicillin
B. cephalexin
C. Metronidazole.
219. Women e vaginal discharge offensive fishy odour revealed clue test d.d
A-Bacterial vaginosis
B-Trichomonas vaginosis
220. Female with vaginal discharge + nucleic acid amplification test (+ve) , Dx?
Gonorrhea
223
Glory 2019 ..
225. Pregnant women not complaining of anything, her urine analysis shows
nitrate, leucocytes, then culture revealed 50.000 organisms what is the dx?
A.cystitis
B.pyelonephritis
asymptomatic bacteriuria
N.B She must be treated.
224
Glory 2019 ..
232. Female came to routine visit and investigation , found high TSH + normal
T3 T4 what the diagnosis ?
a. Primary Hyperthyroidism
b. Primary hypothyroidism
c. Secondary hyperthyroidism
d. Subclinical hypothyroidism
233. Female came to clinic with hx of post coital bleeding , and you do
examination with speculum you found mass in cervix ( picture of mass ) what
the approach for her ?
225
Glory 2019 ..
234. Lady 22yrs with family HX of breast ca came to you with Right lower lateral
mass worsen ,with menstertion , you do biopsy found hyperplasia papillary
adenoma no pain , what you do ?
a. Tell the pt to follow up and came after 6 month with mammography
b. Excesional mass
c. Partial mastectomy
d. Total mastectomy
235. Pregnant lady and her kid started to go to shcoold, she is afraid that he will
affect her what vaccine she should take?
Influenza
236. Picture of anus with Cauliflower, for lady who do multiples unprotected sex
condylomata.
226
Glory 2019 ..
241. Couple come by infertility after investigation done show bilateral fallopian
tube obstruction what will do
A. Tell wife
B. Tell husband
C. Tell couple
242. Pregnant fear that her son may transfer an infection from his school Which
vaccine to give?
nfluenza
243. Preganat c/o sever abd pain and uterine cotractions she was given 6mg Mg
sulfate and the e contractions become normal then decrease the dose to 4mg
Then complained of shortness of breath What to do?
A. give her Ca gluconate
B. Stop Mg sulfate
C. measure serum Mg sulfate
D. fluid
** after stop Mg sulfate give the Ca gluconate
244. pregnant during ANC do 50gm glucose test and find high glucose level She
is more laible to which type if infection?
A. candida
B. trichomonus
C. bacterial vaginitis
D. atrophic vaginintis
227
Glory 2019 ..
246. pregnant in early 2nd trimester c/o painful cx bleeding what to do?
A. pap smear
B. coloposcopy
C. biopsy
D. transvaginal US
** Threatend abortion, Transvaginal US
254. female in reproductive age , come by an ambulance (RTA ) What 1st thing u
have to do?
check for pregnancy
228
Glory 2019 ..
Features: vaginal spotting (mild bleeding every 4-5 days), vaginal soreness, burning,
dryness, redness, thin vagina with few or non fold, pH alkalosis > 5, periodic petechiae
Treatment: topical vaginal estrogen
229
Glory 2019 ..
264. 28 years old female with normal pap smear 3 years old what to do :
A. Nothing
B. Pap smear with cytology
230
Glory 2019 ..
270. A 32-year-old woman sees her physician because she has noticed milk- like
discharge from her breasts the past 4 weeks. She also states that she has not
menstruated in 2 months. The examination reveals galactorrhea but is
otherwise normal.
Here our workup will be; initially measuring serum Prolactin, if high so next step is MRI
Pituitary to confirm diagnosis
** Always exclude states such as pregnancy, lactation, hypothyroidism and medications
before starting the workup of hyperprolactinemia. Prolactinomas may co-secrete growth
hormone (GH).
272. Pregnant lady in week 13 GA, came with bleeding and suspected she have
an infection what test you do?
A. Pap smear
B. Biopsy
C. Colposcopy
D. invasive
231
Glory 2019 ..
Section 4
Surgery …..
232
Glory 2019 ..
1. .29 Years old male did sleeve gastrectomy 6 days ago, came to the ER
complaining of persistent vomiting since operation, on exam nothing
significant, ABG normal ( there were numbers ) , US normal. Best
management?
A.laparoscopy
B.Admissions
C.laparotomy
D.reassure
N.B: Dumping syndrome: postprandial symptoms like epigastric fullness, pain, vomiting.
It's treatment is only small multiple low charbohydrate, low fat and high protein meals +
avoiding liquids with meals.
N.B: If there is persisting vomiting for several months aftergastric surgery so think of
narrowing or stricture and next is gastroscopy.
2. 21 year old male complains of pain around umbilicus then traveled to rt iliac
quadrant tenderness ,reboundtenderness,positiverovsing sign Vomiting,
constipation. What is the DX?
A.polyposis
B.Appendicitis
C.intestinal obstruction
D.intussusception
233
Glory 2019 ..
4.Case Scenario for pt with Mitral Valve prolapse, the question is WOTF is true
regarding the surgery ?
Don’t give the Ab before surgery
5.TOF
7.Man in ER with pain in the right upper abdomen with vomiting ultrasound
shows thick wall of gallbladder and fluid accumulation around management
A - lap cholecystectomy after 12 weeks
B - percutaneous drain
C - open cholecystectomy
234
Glory 2019 ..
11.Patient with fever right upper abdominal pain +ve serology of echinococcus (
hydatid disease) **picture management
A- albendazole
B- cyst
C- Laparoscopiccystectomy
D - percutaneous drainage
12. Pt with appendectomy what is the most common infection post op?
A- fragilisbacterial
B- staph
C- shigella
D- pseudomonas
The organisms most frequently isolated from the wound after appendectomy are
Bacteroides (especially B. fragilis)
N.B: Thyroid surgery is a clean procedure so no need for prophylactic ABx N.B: Valvular
disease is not indicated for endocarditits ABx prophylaxis.
14.pt with constipation for 2 days suffer from sever anal pain ex at 6 & 12 o’clock
A- not laceration
B- internal sphincter abscess
C- anal fissure
D- fistula
N.B: Most common site of anal fissure is 6 (posterior) then 12 o'clock N.B: TTT of anal
fissure by lateral internal sphicterotomy
235
Glory 2019 ..
236
Glory 2019 ..
B- anal fissure.
N.B: features of anal fissure are: visible tear around anus, skin tag or lump near tear,
sharp anal pain related to defecation, streaks of blood on stools, burning or itching in
anal area.
N.B: Best for hernia is open surgery except: female for cosmetic reason + bilateral +
obese so here it is better for lap surgery.
31-Pt had chest trauma nad 3⁄4 rib fracture , paradoxical chest movement ?
A-chest tube
B-needle aspiration
237
Glory 2019 ..
39-a pt rescued from a burning building ....he is conscious with singed nasal
hairs
A- intubation
B- respiratory support
42-Trauma to knee can’t fix tibia to femur with severe pain and swelling which
ligament affected tibial lig
A-ACL
B- PCL
238
Glory 2019 ..
43-pt with fever and upper right pain with positive echinococus
A- Albendazol
B- Cystderoofing
C- Laprascopiccystectomy
D- Perctunouse drain
44-man inthe ER with pain in right lower quadrant of abdomen, nausea and
vomiting. U/S show thick wall gall bladder with fluid surrounding. Management?
A.Lap cholecystectomy after 12 weeks
B.Percutaneous drain
C.Open cholecystectomy
46- Man with HTN, labs show normal adrenal gland, but CT shows mass with lipid
change. Management?
A- adrenalectomy
B- MRI adrenal
C- observation
48-Before adrenal gland operation, what should you give the patient?
Hydrocortisone
239
Glory 2019 ..
53-Asd
58- Elective surgery to pt ,pt don’t want to know details but anesthesiologist
insist to tell her. What to do?
A. Tell pt anesthesiologist decision
B. Reschedule surgery until see another anesthesiologist
59- thyroid-Single nodule on one lobe of the thyroid. What is the surgical
treatment?
A. total thyroidectomy
B. subtotal
C. left lobe thyroidectomy
Depends
If toxic nodule , do Thyroid scan then : total if cold, hemi if hot
If non toxic nodule, do FNA then, if malignant( medullary ) so total
61- epigastric pain, tenderness, pain radiated to back, x ray show air under
diaphragm, what the most likely diagnosis?
A. Perforated duodenal ulcer
B. Pancreatitis
C. Gastric ulcer
240
Glory 2019 ..
62-Patient fell from 50 feet exam decreased breath sounds on left side CXR
picture of pleural effusion on left side what is the treatment?
A- chest tube -hemothorax-
B- thoracotomy
& I dont remember.
63-pt did laparoscopic cholecystectomy and you found a large mass in the
stomach, what you will do :
A-cholycystectomy
B-stop the procedure
C-resection of mass
D- cholysytoctomy and resection of mass
66- pt with one leg pain progress over month what you will ask in hx:
A-hx of truma
B-family hx of same complain
68- consultant at OR and ask to prepare right kidney for surgery but the intern
read in notes it is left kidney what is the most appropriate action by intern?
A- inform the consultant
B-leave the OR
C-go out to re read the note
241
Glory 2019 ..
70- pt with pruritus and jundice and abd pain he k/c of Ulcerative colitis what is
the dx?
A-primary biliary cholangitis
B-primary scleorsing cholangitis
71- (55 years old) female her last mammogram was normal when she will do the
next after?
A-1 year
B-2year
C-3-year
D-4 year
72- 23 y female with symptoms of IBS and she had concer because his uncle had
colon cancer at 60 y what dx?
A-IBS
73- old pt with DM and Ischemic heart disease and gas severe abd pain what dx?
A-mesentric ischemia
B-appendicitis
74-pt with rt knee pain he has epigastric burning what you will give ?
A-ibuprofen
B-paracetamol
75- pt came to the ER can’t pass stool since 2 days and has sever pain in exam
you can’t do PR bc of pain ,pt did hemorrhoidectomy 12 months ago what is the
cause ??
A-anal stenosis
B-anal fissure
Explanation: Anal stenosis is a rare but serious complication of anorectal surgery, most
commonly seen after hemorrhoidectomy
242
Glory 2019 ..
80- Pt with with otitis external after swimming in pool which Abx to give?
Explanation
The most common cause of acute otitis external is pseudomonas bacteria infection,
fungal candidiasis by candida albicans is usually resulted from over treatment with
topical antibiotic, TTT of bacterial otitis external is by antipseudomonal like
Ciprofloxacine or Ceftazidime otic drops while TTT of fungal otitis external is by
repeated debridement and topical anti-fungal agents like Gentian violet or Mycostatin
powder)
^ Toronto
81- Pt with hx of trauma to abdomen came after several days with purulent
discharge what to do?
A- Ctabdomen.
B-Lapratomy
If stable >> CT
Unstable >>lapratomy
83- Pt with gun shout to chest, underwent chest tube which drained 2L of blood.
What is the next step?
A- Pleural tab.
B- Embolization.
C- Emergency Department Thoracotomy!!
84- Dx pt with painful progressing swelling with discharge at his neck what is the
dx?
A- Cellulitis.
B- Abscess.
243
Glory 2019 ..
86- Alcoholic patient with sever epigastric pain radiated to back, nausea and
vomiting, elevated lipase and amylase. And obstructive pattern of LFT. Dx?
A. Pancreatitis
Explanation :for staging , breast cancer Mets ( Bone , lung ,liver, brain )
89- Case of painless freely mobile breast lump not related to menstruated
changes. What is the diagnosis?
A- Fibroadenosis.
B- Fibroadenoma.
C- Fat necrosis.
D- Breast cancer.
90- pt with hernia came with acute swelling , redness , tender , not reducible
Which complication of hernia is this
Iirdeuceable
Incarcerated
Strangulated
92- adult with epigastria pain us showed several narrowing of biliary duct
LFT obstructive pattern what is the dx
Primary sclerosing cholangitis
244
Glory 2019 ..
93- pt presented with appendicitis ssx , us should obesity in the pelvis what to do
next
ct abdomen
94- mid age man with appendoceal after conservative ttt recovered and discharge
what to do next
- - no thing
- Laproscopic appendectomy in 12 wks
- Open appendectomy in 12 wks
95- pt after appendectomy stble with mild pain, should minmal collection
- Conservative mx
- - re open
- - aspirate
98- case of bowel obstruction imaging showed y shape shadow and enlarge
colon what is thenext step ?
A- Enema
B - CT abdomen
99- pt with stab wound to chest , SOB, I think low o2 saturation and ssx of
pneumothorax next step ?
- If dullness on percussion so hemo so chest tube
- If hyperresonant so tension pneumothorax so needle then tube
245
Glory 2019 ..
102- what is the most common GIT malignant associated with peutz-
jegherssyndrome ?
A- stomach
B- pancreas
C- colon
D- liver
104- patient comes with neck swelling, moving with deglutition located hyoid
area, what’s?
a. Thyroglossalcyst
b. Thyroidcancer
c. CysticHygroma
105- 21 years old women presents to the gynaecology Clinic with a mass in the
left breast, she discovered this mass while showering, her last menstrual peroid
was 10 days ago, there’s no family history of breast cancer.
On physical examination, you palpate a 3 cm, rm, non tender mass in the upper
lateral quadrant of the left breast, the mass is smooth, well circumscribed and
mobile there’s no skin changes or nipple discharge or axillary lymphadenopathy.
Which of the following is the most likely diagnosis in this woman?
a. Fibroadenoma
b. Fibrocysticchange
c. Intraducalpapailoma
d. Lobularcarcinomainsitu
106- lady with a mass in left upper quadrant of the breast, aspiration was yellow
uid without masses, this mass completely disappears with aspiration, what’s the
diagnosis?
a. Phyllod
b. breastcancer
c. broadenoma
d. fibrocysticdisease
246
Glory 2019 ..
109- A 37-year old women presented to the doctor with intermittent bloody
nipple discharge from the left breast. On examination there is no palpable
mass. Which of the following is the most likely diagnosis?
A. Fibroadenoma
B. Fibrocystic changes
C. Intraductal papilloma
D. Mastitis
110-A 32- year- old female presented to her physician with feel small lump
in her right breast , On Examination and investigation normal, what is the
best asdvise can tell her to do self-breast exam every?
A. 1 week
B. 2 month
C. 3 month
D. month
247
Glory 2019 ..
112- patient lady noticed lump for three months the mass freely mobile no
discharge not related to menstrual, what is the diagnosis?
A. cystic
B. duct ectesia
C. ductal papilloma
D. Fibroadenoma
113- Which of the following is the best treatment for a lung abscess?
a. Ciprooxacin
b. Clindamycin
c. Macrolides
d. Piperacillin
114- Female work as an actress, present with mass that increases before
menses, after taking FNA the result is yellow stain with no refilling again
what is the diagnosis?
A. Fibrocystic breast disease
B. follicular cancer
C. intraductal papilloma
D. phyllodes tumor
248
Glory 2019 ..
117- patient presents to the hospital with bloody nipple discharge, what is
the most likely cause?
A. Intraductal papilloma
B. ductal
C. lobular carcinoma in situ
D. papillary
119- patient noticed lump for three the mass freely mobile, no discharge
not related to menstrual , what is the diagnosis ?
A. cystic
B. duct ectesia
C. ductal papilloma
D. Fibroadenoma
Explanation:
Needle :2
Chest tube : 5
249
Glory 2019 ..
Liver biopsy :7
Thoracosentesis : 8-10
124- Patient with breast cancer, she is having bleeding from breast with redness
,on routine check 2x3 cm then became 3x6cm.What do you suspect?
A. bad prognosis
B. inammatory process
C. malignant cancerous change
D. metastasis precess
125- In patient post mastectomy, they do for her reconstruction from the rectus
muscle. what is the vessels maybe injured or effected?
A. Inferior epigastric artery
B. Superior epigastric artery
C. intercostal artery
D. internal thoracic artery
250
Glory 2019 ..
126-A 70 year old patient is complaining of face ushing and head heaviness in
the morning
that's relieved during the day. He has been smoking for 30 year.s CXR shows
bilateral hilar masses. What is the diagnosis?
A. COPD
B. Hodgkin's lymphoma
C. Lung cancer
D. TB
130-30 years old female dancer with breast mass that disappeared by
aspiration, What is your suspect?
A. cystic –fibroadenosis-
B. duct ectesia
C. Fibroadenoma
D. phylloid
251
Glory 2019 ..
132-A 37-year old women presented to the doctor with intermittent bloody
nipple discharge from the left breast. On examination there is no palpable
mass. Which of the following is the most likely diagnosis?
A. Fibroadenoma
B. Fibrocystic changes
C. Intraductal papilloma
D. Mastitis
133-A 32- year- old female presented to her physician with feel small lump
in her right breast , On Examination and investigation normal, what is the
best asdvise can tell her to do self-breast exam every?
A. 1 week
B. 2 month
C. 3 month
D. month
135-patient lady noticed lump for three months the mass freely mobile no
discharge not related to menstrual, what is the diagnosis?
A. cystic
B. duct ectesia
C. ductal papilloma
D. Fibroadenoma
For 6 weeks
252
Glory 2019 ..
Needle :2
Chest tube : 5
Liver biopsy :7
Thoracosentesis : 8-10
253
Glory 2019 ..
143- Unmarried young lady , 20 yr old, with bilateral abdominal pain, she is a
waitress at a resort, very vague sexual history, no bleeding, no significant and
examination….no fever, missed last month periods….diagnosis.
a. ovarian torsion
b. salphingitis
c. ruptured ectopic pregnancy
d. endometritis and adnexal masses
254
Glory 2019 ..
146- 58 yr old woman, h/o epigastric pain, for 2 months, no rebound tenderness,
no abdominal mass, no bruit heard over abdominal area, history of mild
discomfort over chest since 6 months ECG of ST elevation in 2, 3, AvF given….
a. MI > inferior
b. peptic ulcer disease
c. abdominal aortic aneurysm
d. coarctation of aorta.
147- pt with thyroid function test… TSH 400 T3 normal T4 normal, diagnosis…
a. sub clinical thyroiditis
b. primary hyperthyroidism
c. primary hypothyroidism
d. goitrogenic cancer
149- pt with scaling skin lesion in the extensor surface of the forearm ( with
picture), your advice:
a- avoid trauma.
b- avoid sunlight
c. apply sunscreen
d. green tinged make up
150- 55year old patient with dysphagia for solids with a previous history of reflux
A. Carcinoma oesophagus
B. Stricture
C. Scleroderma
D. Achalasia
255
Glory 2019 ..
152- Patient came for assessment after colle’s fracture by falling on outstretched
arm with minimal trauma, what is the appropriate test to check for bone density?
A- VIT D
B- Calcium level
C-X RAY hip and pelvis
D-Dual energy x ray absorptiometry
256
Glory 2019 ..
156- Patient comes with neck swelling, moving with deglutition located hyoid
area. What is diagnosis?
a) multi nodular goiter
b) Thyroglossal cyst
c) Thyroid cancer
d) Cystic Hygroma
157- pt with enlarged parotid gland with secretions increased over time, best
initial investigation
A. USG
B. CT
C. X-ray
D. MRI
Best initial is US
Confirm is MRI
158-Q about Slipped capital femoral epiphysis Clear scenario and X ray
Obese In boys more than female txx:surgical
257
Glory 2019 ..
159-Which nerve carries the referred painof the parotid to the ear?
A. Vagus
B. Facial
C. Auriculotemporal
D. Trigeminal
161-ase scenario about a patient who had an injury in his right knee, the doctor
applied a valgus stress to examine which ligament
A- Anterior cruciate ligament
B- Posterior cruciate ligament
C- Fibular collateral
D- Tibial collateral
258
Glory 2019 ..
165-Soldier with pain at medial scle of foot with Hx of walking for long distance,
which ligament is affected?
Spring Ligament
166-Mutiorgan failure after trauma lab [ high phosphate] which organ can cause
this ?
Kidney
169-17 year old runner (Athlete) when preue of knee cab kid pain Dx ?
osgood Schlatter
259
Glory 2019 ..
175-Female with problem in chest go for mastectomy we will take from abdominal
muscle for
graft witch artery is response for it ?
A. superior epigastric
B. Inferior epigastric
177- Pt with hyperaldosteronism [normal renin -BP -aldosterone] what's the cause
in the adrenal cortex for this?
Glomerulosa.
Aldosterone, the main mineralocorticoid hormone, is a steroid hormone produced by the
zona glomerulosa of the adrenal cortex in the adrenal gland.
178- Case of cardiac tamponade, hypotensive, heart voice not clear, trauma,
distend
venous neck, next step?
Pericardiocentesis.
Triad of cardiac tamponade are muffled distant heart sounds + distended neck veins +
hypotension.
179- Pt with cholecystectomy develop pain in the angle of the mouth what u wll
do?
Paracetamol.
180- U did appendictomy and you found it not inflamed you remove it and closed
what the next?
A. Tell the pt.
B. Tell the ethics.
C. Tell the surgery head.
181- You are doing appendictomy u found bus and fluid in the RIF. What u will
do?
A. Remove the fluid and appendix.
B. Remove the appendix.
260
Glory 2019 ..
182- Pt with AAA 4.5 cm, have cholecystectomy, vascular surgery councled what
he wll say?
A. proceed the surgery and folw up with US.
B. PROCEED THE SURGRY FOLOW WITH CT.
C. CT THEN DO THE SURGERY.
183- Pt with recurent abdominal pain us show dilated intrahepac duct T 38 pain
increase with morphin, LFT Normal ALP high what is diagnosis?
A. cholelithiasis.
B. choleangitis.
C. cholecystitis.
184- Pt have epigastric pain after vomiting CXR show some pleural effusion what
is diagnosis ?
rupture esophagus.
185- Pt with Abdominal mass after lefting heavy object mass not change with
cough
what is diagnosis?
A. rectus muscl heamatoma.
B. hernia.
186- Pt with testicular pain and vomiting scrotum is red what u wll do?
A. US.
B. urgent surgry.
187- Patient fall in out stretched hand ...pic of open colles fracture what you
will.do next
A. Open reduction
B. close reduction
C. debridement with open reduction
rule of 2's; 2% of the population, within 2 feet of the ileocecal valve, 2 inches in length,
261
Glory 2019 ..
191- Patient with breast Ca, polyuria and increase thirst, she has also
Hypercalcemia: what will u give her
A. Normal saline
B. Furosemide
C. Amlodipine
192- Patient with unilateral knee pain and urethral discharge of pus. Negative
culture of the pus:
A. Riter syndrome
> triad
- Since cultures are negative it’s chlamydia
262
Glory 2019 ..
193- Parklond formula, how much fluid are given in first 8hrs for anterior trunk
and circumferenal le= upper and le= lower limbs: 6.3
194- patient with Hight parathyroid hormone and hypercalcemia, what u will do
next:
Look for parathyroid adenoma
TSH, T3 and T4
197- patient with case of otitis media and erytham on mastoid bone, what’s the
treatment:
A- Amoxicillin
B- Cefoxaim
263
Glory 2019 ..
198- 19 yrs old lady with Diarrhea for 10 months, wt loss, postprandial
periumbilical pain. Diarrhea sometimes bloody
A- Crohn’s
B- Peptic ulcer
C- Chronic Pancreatic
199- Pt with cholecystectomy develop pain in the angle of the mouth what you
will do?
Paracetamol
200- U did appendectomy and you found it not inflamed you remove it and closed
what the next
• Tel the pt
• Tel the ethics
• Tell the surgery head
201- Your doing appendectomy u found bus and fluid in the RIF what u wl do
• Remove the fluid and appendix
• Remove the appendix
202- PT WITH persistent abdominal pain US show dilated intrahepatic duct temp
38 pain increase with morphing LFT Normal ALP high what is diagnosis
• cholelithiasis
• choleangitis
• cholecystitis
203- Pt with AAA 4,5 cm, have cholecystectomy, vascular surgeon consult, what
he will say?
• Proceed the surgery and follow up by US
• Proceed the surgery and follow up with CT
• Ct the do the surgery
206-management of cholelithiasis
264
Glory 2019 ..
Elective Cholysystectomy
208-man with both lower limbs burn, ask about fluid replacement
265
Glory 2019 ..
212-Patient with complex reagonal pain after surgery to carpal tunnel . Treatment?
NSAID
216- Male with Gun shots. Inter lateral to left nipple .and exit under the scapula.
The patient collapsed in ER. What to do?
A-Iv.fluid
B-Pericardiocentesis
266
Glory 2019 ..
220-Elderly with hem a you're a.cystoscopy done and show in the uppermost of
bladder mass? .
A-Transitional cell carcinoma
B-squamous cell carcinoma
223- 26 yrs male did appendectomy then was discharged next day as he was wel,8
hours later he came back to hospital with mild pain at site of incision, US
reaveled colection 2cm x2cm ,HB 11 TLC 7.9 bp 110/70 p.76 RR 16
How would you do
A.Exploration
B.observation
N.B: Wound exploration is the best ttt of any postoperative surgical site infection.
224- 35 years old male, I don't remember his complain, on colonoscopy traveled
multiple polyps, your diagnosis?
A- familial polyposis
By cancer
226- 32 years old male , his left leg is pallor and cold , no hair , your diagnosis ?
A- atrial embolism
B- atrial thrombosis
C- DVT
N.B: Trophic changes like hair loss support atherosclerosis and thrombosis
228- Most common site of rodent ulcer (Basal cell carcinoma of the skin)?
267
Glory 2019 ..
A- foot
B- arm
C- face
D- shoulder
230- Pt c/o right upper abdominal pain in the right side colicky in nature, radiate to
back, in CT found stones (no write the diameter ) in gall bladder, with normal
Galbladder?
1- Elective Cholestectomy
2- Ursodeoxy colic acid
231- Patient was playing football and then suddenly he heardapop, he is not able
to walk on his tip toe ,what ligament is injured?
A. Quadriceps
B. Calcaneal tendon
233- Patient feel scalf pain while waking with loss of hair and the index is 0.8,
what is the diagnosis?
A. Limb ischemia
B. DVT
C. Venous insufficiency
237- DM pt came to ER with DIC on heparin, past hx of claudication, atrial fib After
DIC what to stop?
A. heparin
268
Glory 2019 ..
B. insulin
C. furosmide
239- 26 yrs male did appendectomy then was discharged next day as he was well ,
8hours later he came back to hospital with mild pain at site of incision, US
reaveled collection 2cm x2cm , HB 11 TLC 7.9 bp 110/70 p. 76 RR 16 , How would
you do
A. Exploration
B. observation
C. CT
D. Need le suction
242-Pt did surgery in colon, came after 1 week with low grade fever, what you’ll do
?
A- MRI
B- CT
C- exploration
269
Glory 2019 ..
244-Pt came with lipoma in neck then discover thyroid nodule (suspicious)
What you’ll do ?
A- excision nodule
B- hemi thyroidectomy
247-Same case but with intact air entry & percussion dull
A- cardiac tamponade
270
Glory 2019 ..
254-Pt c/o of S.O.B on ex there is muffled heart sound and dilated ventricle what
is the diagnosis
A/core pulmonale
B/pulmonary edema
C/Cardiac tamponade
255-Pt c/o thyroid enlargement (multi nodular or diffuse l not remember) and there
is two lumps One is smal1×2 and other larger 3×4 what to do
A/FNA from largest one
B/FNA from both lumps
C/Excisional biopsy
257- 70 years old complain of fresh blood from rectum and when to do endoscopy
you found
polyps what is the management :
A/ Reassurance
B/ tell him to come for follow up after 6 wks
C/ do resection of polyp
D/ resection of rectum ( something like that )
271
Glory 2019 ..
258-pt came to the ER and complained of burn in his tow lower limbs
his wt 70 kg what is the amount of fluid must you give him
A/ ringer lactate 2500 ml at 1 st 8 hrs and 2500 ml at second 1 6 hrs
B/ NS 5000 ml at 1 st 8 hrs
261-Pt with enlargement of Thyroid, large after in thyroid scan, what the second
option
263- Patient with bag of worms in the scrotum, what will you do ?
Dopler
272
Glory 2019 ..
270- Spinal stenosis case. Relieve after leaning forward pain decreases
273- post operative of appendicectomy came with vague Abd pain o/x mild
tenderness CT shows retrocecal collection 2×2 what to do?
274- -post lap cholecystectomy developed high grade fever with shills CT
subhepatic collection what to do
Depends on the size if small abx and observe, If drainage add abx
275- with lower abd pain o/px mass, lower abdomen tenderness mainly RIF with
fullness, pregnancy test negative?
a)acute cystitis
b)ovarian torsion
c)rupture cyst
d)appendicitis
276- pt c/o thyroid enlargment (multi noduler or difuse l not remember) and there
is two lumps ones small 1×2 and other larger 3×4 what to do?
A/ FNA from largest one
B/ FNA from both lumps
C/Excisional biopsy
273
Glory 2019 ..
277- One case of pain on the thenar aspect of the hand after a Surgery n the
patient been , seen by orthopaedic.
A.give analgesics.
B.opiot.
C.physioyherapy. ??
D.codeine
278-After 12 hours of gastric slap surgury, pt Came with upper abdominal pain,
every thing normal?
Increase analgesia
Laporotomy
280- Patient with metastatic breast ca present with SOB , distended neck vein,
apex beat not palpable. distal heart sounds , BP 70/20. The best step to conrm the
diagnosis?
A. IV fuid & urgent echo
B. IV furosemide
C. Pericardiocentesis D. chest tube
281- Patient with the breast mass, which of the following indicates that the mass
is cystic (or no need for reevaluation)?
A. Bloody nipple discharge
B. Clear fluid with reappearance of the mass
C. FNA showing clear fluid on aspiration and disappearance of the mass after
D. fatty tissue appearance
282- 41-year-old Lady has breast mass since 2 weeks, upon examination it hard
and fixed, what is "most " accurate diagnostic modality?
A. Lactography
B. MRI Breast
C. Mammography
D. US
274
Glory 2019 ..
192-
-
1497-
275